Está en la página 1de 182

GMAT Critical Reasoning - Everything you need to know

This slideshow features screenshots from GMAT Prep Nows


entire Critical Reasoning module (consisting of 38 videos).
It covers everything you need to know, and it includes 24
practice questions.

www.GMATPrepNow.com

500+ videos

800+ practice questions

step-by-step learning guide

All you need. All for free

GMAT Critical Reasoning - Everything you need to know

Note: since these slides are just snippets of a full-length video


course, there may be times when youre unable to glean all the
relevant information from a particular screenshot.
If, at any time, youd like to watch the entire video on a certain
topic, just click on the link at the top of that page, and youll be
taken that that particular video.

www.GMATPrepNow.com

500+ videos

800+ practice questions

step-by-step learning guide

All you need. All for free

GMAT Critical Reasoning - Everything you need to know

If you enjoy this unique learning format,


let us know, and well add similar
resources to our SlideShare page

500+ videos

800+ practice questions

step-by-step learning guide

All you need. All for free

Introduction to Critical Reasoning

500+ videos

800+ practice questions

(watch the entire video here)

step-by-step learning guide

All you need. All for free

Introduction to Critical Reasoning

(watch the entire video here)

This lesson features GMAT questions from the GMAT mini-test.


GMAT questions are the property of the Graduate Management
Admission Council and are used here with their permission.

500+ videos

800+ practice questions

step-by-step learning guide

All you need. All for free

Introduction to Critical Reasoning

(watch the entire video here)

About 1/3 of Verbal questions (13 to 15 questions)


Batches of 2 or 3 questions

Approximately 2 minutes per question


Test your ability to reason effectively

500+ videos

800+ practice questions

step-by-step learning guide

All you need. All for free

Introduction to Critical Reasoning


The average normal infant born in the United
States weighs between twelve and fourteen
pounds at the age of three months. Therefore,
if a three-month-old child weighs only ten
pounds, its weight gain has been below the
United States average.

(watch the entire video here)

Passage

Which of the following indicates a flaw in the


reasoning above?
A) Weight is only one measure of normal
infant development.
B) Some three-month-old children weigh as
much as seventeen pounds.
C) It is possible for a normal child to weigh
ten pounds at birth.
D) The phrase "below average" does not
necessarily mean insufficient.
E) Average weight gain is not the same as
average weight.

500+ videos

800+ practice questions

step-by-step learning guide

All you need. All for free

Introduction to Critical Reasoning


The average normal infant born in the United
States weighs between twelve and fourteen
pounds at the age of three months. Therefore,
if a three-month-old child weighs only ten
pounds, its weight gain has been below the
United States average.
Which of the following indicates a flaw in the
reasoning above?

(watch the entire video here)

Passage

Question stem

A) Weight is only one measure of normal


infant development.
B) Some three-month-old children weigh as
much as seventeen pounds.
C) It is possible for a normal child to weigh
ten pounds at birth.
D) The phrase "below average" does not
necessarily mean insufficient.
E) Average weight gain is not the same as
average weight.

500+ videos

800+ practice questions

step-by-step learning guide

All you need. All for free

Introduction to Critical Reasoning


The average normal infant born in the United
States weighs between twelve and fourteen
pounds at the age of three months. Therefore,
if a three-month-old child weighs only ten
pounds, its weight gain has been below the
United States average.
Which of the following indicates a flaw in the
reasoning above?

(watch the entire video here)

Passage

Question stem

A) Weight is only one measure of normal


infant development.
B) Some three-month-old children weigh as
much as seventeen pounds.
C) It is possible for a normal child to weigh
ten pounds at birth.

Answer choices

D) The phrase "below average" does not


necessarily mean insufficient.
E) Average weight gain is not the same as
average weight.

500+ videos

800+ practice questions

step-by-step learning guide

All you need. All for free

Introduction to Critical Reasoning

(watch the entire video here)

About 1/3 of Verbal questions (13 to 15 questions)


Batches of 2 or 3 questions

Approximately 2 minutes per question


Test your ability to reason effectively
Arguments consist of premises and a conclusion
Conclusion: what the author is trying to convince you of
Premises: the evidence used to support the conclusion

500+ videos

800+ practice questions

step-by-step learning guide

All you need. All for free

Introduction to Critical Reasoning

(watch the entire video here)

Strength of an Argument: How well the conclusion follows


from the premises

All men are mortal. Socrates is a man.


Therefore, Socrates is mortal.

Deductive: the conclusion


is guaranteed

Premise: All men are mortal


+ Premise: Socrates is a man
Conclusion: Socrates is mortal

For the past 3 days, Gary has arrived late


for work. This morning, while Gary was
driving to work, his car got a flat tire.
Therefore, Gary will be late for work today.

Inductive: the conclusion


is not guaranteed

Premise: Gary late for last 3 days


+ Premise: Flat tire this morning
Conclusion: Gary will be late today

500+ videos

800+ practice questions

step-by-step learning guide

All you need. All for free

Introduction to Critical Reasoning

(watch the entire video here)

Strength of an Argument: How well the conclusion follows


from the premises
For the past 3 days, Gary has arrived late
for work. This morning, while Gary was
driving to work, his car got a flat tire.
Therefore, Gary will be late for work today.
Strengthen
For the past 53 days, Gary has arrived
late for work. This morning, while Gary
was driving to work, his car got a flat tire.
Therefore, Gary will be late for work today.
For the past 53 days, Gary has arrived
late for work. This morning, while Gary
was driving to work, his car got 4 flat
tires, and his engine exploded.
Therefore, Gary will be late for work today.

500+ videos

800+ practice questions

Strengthen

step-by-step learning guide

All you need. All for free

Introduction to Critical Reasoning

(watch the entire video here)

Strength of an Argument: How well the conclusion follows


from the premises
For the past 3 days, Gary has arrived late
for work. This morning, while Gary was
driving to work, his car got a flat tire.
Therefore, Gary will be late for work today.
Weaken

For the past 3 days, Gary has arrived late


for work. This morning, while Gary was
driving to work, his car got a flat tire.
Garys house is 1 block from work.
Therefore, Gary will be late for work today.

500+ videos

800+ practice questions

step-by-step learning guide

All you need. All for free

Introduction to Critical Reasoning

(watch the entire video here)

Strength of an Argument: How well the conclusion follows


from the premises
For the past 3 days, Gary has arrived late
for work. This morning, while Gary was
driving to work, his car got a flat tire.
Therefore, Gary will be late for work today.
The conclusions in most arguments are not guaranteed
Most arguments can be strengthened or weakened by
adding/altering premises

500+ videos

800+ practice questions

step-by-step learning guide

All you need. All for free

Introduction to Critical Reasoning

(watch the entire video here)

Accept all premises as true!

For the past 3 days, Gary has arrived late


for work. This morning, while Gary was
driving to work, his car got a flat tire.
Therefore, Gary will be late for work today.
Strength of an Argument
How well the conclusion follows
from the premises

500+ videos

800+ practice questions

step-by-step learning guide

All you need. All for free

Introduction to Critical Reasoning

(watch the entire video here)

Accept all premises as true!

All pigs can fly. Mount Everest is a pig.


Therefore, Mount Everest can fly.

500+ videos

800+ practice questions

step-by-step learning guide

All you need. All for free

Introduction to Critical Reasoning

(watch the entire video here)

Accept all premises as true!

All pigs can fly. Mount Everest is a pig.


Therefore, Mount Everest cannot fly.

500+ videos

800+ practice questions

step-by-step learning guide

All you need. All for free

Introduction to Critical Reasoning

(watch the entire video here)

About 1/3 of Verbal questions (13 to 15 questions)


Batches of 2 or 3 questions

Approximately 2 minutes per question


Test your ability to reason effectively
Arguments consist of premises and a conclusion
Conclusion: what the author is trying to convince you of
Premises: the evidence used to support the conclusion
Assumption(s): unstated premise(s) necessary to reach conclusion

500+ videos

800+ practice questions

step-by-step learning guide

All you need. All for free

Introduction to Critical Reasoning

(watch the entire video here)

For the past 3 days, Gary has arrived late


for work. This morning, while Gary was
driving to work, his car got a flat tire.
Therefore, Gary will be late for work today.
Assumptions: Garys history of lateness will be
repeated today
The time needed to change tire will
exceed the time before work starts
Gary has no other means by which to
arrive at work on time

500+ videos

800+ practice questions

step-by-step learning guide

All you need. All for free

Introduction to Critical Reasoning

(watch the entire video here)

Premise(s)
+ Assumption(s)
Conclusion

500+ videos

800+ practice questions

step-by-step learning guide

All you need. All for free

Introduction to Critical Reasoning

(watch the entire video here)

About 1/3 of Verbal questions (13 to 15 questions)


Batches of 2 or 3 questions

Approximately 2 minutes per question


Test your ability to reason effectively
Arguments consist of premises and a conclusion
Conclusion: what the author is trying to convince you of
Premises: the evidence used to support the conclusion
Assumption(s): unstated premise(s) necessary to reach conclusion

Strength of an Argument: How well the conclusion follows


from the premises
Accept all premises as true!

500+ videos

800+ practice questions

step-by-step learning guide

All you need. All for free

Dissecting an Argument

500+ videos

800+ practice questions

(watch the entire video here)

step-by-step learning guide

All you need. All for free

Dissecting an Argument

(watch the entire video here)

Every hockey fan I know is nice. I do not


know Judy, but since she is wearing a
hockey jersey, she must be nice.
Summarize:

- the conclusion
- the premises
- any assumptions

500+ videos

800+ practice questions

step-by-step learning guide

All you need. All for free

Dissecting an Argument

(watch the entire video here)

Every hockey fan I know is nice. I do not


know Judy, but since she is wearing a
hockey jersey, she must be nice.
Is this what the
author is trying to
convince me of?
Conclusion: J is nice

500+ videos

800+ practice questions

step-by-step learning guide

All you need. All for free

Dissecting an Argument

(watch the entire video here)

Every hockey fan I know is nice. I do not


know Judy, but since she is wearing a
hockey jersey, she must be nice.
Goal: make your summaries as brief as
possible (while maintaining coherence)
Conclusion: J is nice

500+ videos

800+ practice questions

step-by-step learning guide

All you need. All for free

Dissecting an Argument

(watch the entire video here)

Every hockey fan I know is nice. I do not


know Judy, but since she is wearing a
hockey jersey, she must be nice.

Premise:
Premise:
Premise:
Assumption:
+ Assumption:
Conclusion:

500+ videos

All H-fans I know are nice


I dont know J
J wearing jersey
The H-fans I know are representative
Wearing jersey makes one a fan
J is nice

800+ practice questions

step-by-step learning guide

All you need. All for free

Dissecting an Argument

(watch the entire video here)

Researcher: Two years ago, a wolf pack was


relocated to Bilford Island. Although the local
rabbit population has decreased drastically
since the relocation, the wolves are not to
blame for this decrease. Our study shows that
the unprecedented number of recent rabbit
deaths is due to the myxoma virus.
Summarize:
- the conclusion
- the premises
- any assumptions

500+ videos

800+ practice questions

step-by-step learning guide

All you need. All for free

Dissecting an Argument

(watch the entire video here)

Researcher: Two years ago, a wolf pack was


relocated to Bilford Island. Although the local
rabbit population has decreased drastically
since the relocation, the wolves are not to
blame for this decrease. Our study shows that
the unprecedented number of recent rabbit
deaths is due to the myxoma virus.

Premise-Therefore-Conclusion test
Option A: wolves not to blame therefore virus killed rabbits

Option B: virus killed rabbits therefore wolves not to blame

500+ videos

800+ practice questions

step-by-step learning guide

All you need. All for free

Dissecting an Argument

(watch the entire video here)

Researcher: Two years ago, a wolf pack was


relocated to Bilford Island. Although the local
rabbit population has decreased drastically
since the relocation, the wolves are not to
blame for this decrease. Our study shows that
the unprecedented number of recent rabbit
deaths is due to the myxoma virus.

Premise:
Premise:
Premise:
+ Assumption:

wolves arrived 2 yrs ago


rabbit pop. since
virus caused deaths
wolves didnt contribute to virus

Conclusion: rabbit pop. not wolves fault

500+ videos

800+ practice questions

step-by-step learning guide

All you need. All for free

Dissecting an Argument

(watch the entire video here)

Tips for Identifying Conclusions and Premises


Watch for trigger words that indicate a conclusion
therefore, thus, hence, so, implies, indicates, consequently,
as a result, clearly, accordingly, infer, conclude
Watch for trigger words that indicate a premise
since, because, for, due to, evidence, on the basis of,
given that
Beware of common argument structures
1. Premise, premise, . . . , conclusion
2. Conclusion, premise, premise . . .
3. Conclusion in the question stem

500+ videos

800+ practice questions

step-by-step learning guide

All you need. All for free

Dissecting an Argument

(watch the entire video here)

Until recently, the only fish species living in Chilliwack Lake


was the Gigafish. Last month, however, several Sovkafish were
spotted in the lake. Unlike Gigafish, Sovkafish do not eat
insects; instead, they survive by eating other fish. In other
lakes where Sovkafish exist, their populations are limited by
Dragonfish, which like to feed on Sovkafish.
Which of the following, if true, most effectively challenges the
conclusion that releasing 100 Dragonfish into Chilliwack Lake
will allow the Gigafish in Chilliwack Lake to survive?

Summarize:
- the conclusion
- the premises
- any assumptions

500+ videos

800+ practice questions

step-by-step learning guide

All you need. All for free

Dissecting an Argument

(watch the entire video here)

Until recently, the only fish species living in Chilliwack Lake


was the Gigafish. Last month, however, several Sovkafish were
spotted in the lake. Unlike Gigafish, Sovkafish do not eat
insects; instead, they survive by eating other fish. In other
lakes where Sovkafish exist, their populations are limited by
Dragonfish, which like to feed on Sovkafish.
Which of the following, if true, most effectively challenges the
conclusion that releasing 100 Dragonfish into Chilliwack Lake
will allow the Gigafish in Chilliwack Lake to survive?
Premise:
Premise:
Premise:
Premise:
Assumption:
Assumption:
+ Assumption:

Giga were only fish in lake


Sovkas now in lake
Sovkas eat other fish
Dragons eat Sovkas
Dragons won't eat the Gigas
Dragons won't somehow jeopardize Gigas
100 Dragons is sufficient

Conclusion: Releasing Dragons will let Gigas live

500+ videos

800+ practice questions

step-by-step learning guide

All you need. All for free

Dissecting an Argument

(watch the entire video here)

Tips for Identifying Conclusions and Premises


Watch for trigger words that indicate a conclusion
therefore, thus, hence, so, implies, indicates, consequently,
as a result, clearly, accordingly, infer, conclude
Watch for trigger words that indicate a premise
since, because, for, due to, evidence, on the basis of,
given that
Beware of common argument structures
1. Premise, premise, . . . , conclusion
2. Conclusion, premise, premise . . .
3. Conclusion in the question stem
Premise-Therefore-Conclusion test

500+ videos

800+ practice questions

step-by-step learning guide

All you need. All for free

Common Argument Types

500+ videos

800+ practice questions

(watch the entire video here)

step-by-step learning guide

All you need. All for free

Common Argument Types

(watch the entire video here)

3 Most Common Argument Types


Cause and Effect
Statistical
Analogy

500+ videos

800+ practice questions

step-by-step learning guide

All you need. All for free

Common Argument Types


Cause and Effect

(watch the entire video here)

correlation causation

Premise: Event X occurs


Premise: Event Y occurs
+ Assumption: X is the only possible cause of Y
Conclusion: X causes Y

A recent study reveals that the rate of obesity is


higher among senior citizens who watch more than 8
hours of television per day than among senior citizens
who watch fewer than 8 hours of television per day.
Therefore, obesity among senior citizens is caused by
watching more than 8 hours of television per day.
Weaken

Strengthen

Something else causes Y


Y causes X
X and Y are coincidental

500+ videos

800+ practice questions

More information
Eliminate other causes of Y

step-by-step learning guide

All you need. All for free

Common Argument Types

(watch the entire video here)

Statistical
Premise: Information from sample
+ Assumption: Sample represents entire population
Conclusion: Something about entire population
In a recent survey, participants at a Republicansonly dance competition were given a questionnaire.
Most of the respondents indicated that they enjoyed
singing. Therefore, it can be concluded that most
Republicans are outgoing people.
Weaken

Strengthen

Sample not representative


Conclusion doesnt match stats
Flaw in calculations

500+ videos

800+ practice questions

Sample is representative

step-by-step learning guide

All you need. All for free

Common Argument Types

(watch the entire video here)

Analogy
Premise:
Premise:
Premise:
+ Assumption:

Similarity between X and Y


Similarity between X and Y
Similarity between X and Y
Sharing some means sharing all

Conclusion: Some other similarity exists


Country X is a democratic, tropical country with a population
of 5 million, and Country Y is a democratic, tropical country
with a population of 5 million. Since Country X is
experiencing widespread crop failures, Country Y must be
experiencing widespread crop failures as well.

Weaken

Strengthen

Entities less similar

500+ videos

Entities even more similar

800+ practice questions

step-by-step learning guide

All you need. All for free

Common Argument Types


Cause and Effect
Premise: Event X occurs
Premise: Event Y occurs
+ Assumption: X is the only possible cause of Y
Conclusion: X causes Y

Statistical
Premise: Information from sample
+ Assumption: Sample represents entire population
Conclusion: Something about entire population

(watch the entire video here)

Weaken
Something else causes Y
Y causes X
X and Y are coincidental
Strengthen
More information
Eliminate other causes of Y
Weaken
Sample not representative
Conclusion doesnt match stats
Flaw in calculations
Strengthen
Sample is representative

Analogy
Premise:
Premise:
Premise:
+ Assumption:

Similarity between X and Y


Similarity between X and Y
Similarity between X and Y
Sharing some means sharing all

Conclusion: Some other similarity exists

Weaken
Entities less similar
Strengthen
Entities even more similar

General Critical Reasoning Strategy

500+ videos

800+ practice questions

(watch the entire video here)

step-by-step learning guide

All you need. All for free

General Critical Reasoning Strategy

(watch the entire video here)

Question Types
1. Weaken the Argument
2. Strengthen the Argument
3. Assumption
4. Conclusion/Inference
5. Method of Reasoning
6. Flawed Argument
7. Paradox
8. Evaluation

500+ videos

800+ practice questions

step-by-step learning guide

All you need. All for free

General Critical Reasoning Strategy

(watch the entire video here)

General Strategy
1. Read the question stem to determine the question type
2. Read the passage (argument) and focus on the required
information for that question type
3. Check all answer choices

500+ videos

800+ practice questions

step-by-step learning guide

All you need. All for free

Weaken the Argument Questions

500+ videos

800+ practice questions

(watch the entire video here)

step-by-step learning guide

All you need. All for free

Weaken the Argument Questions


Premise
Premise
Premise
Assumption
+ Assumption

Conclusion

(A) New
(B) New
(C) New
(D) New
(E) New

(watch the entire video here)

Premise
Premise
Premise
Premise
Premise

Goal: Find the answer choice that, when added to


the argument, undermines the conclusion the most.

500+ videos

800+ practice questions

step-by-step learning guide

All you need. All for free

Weaken the Argument Questions

(watch the entire video here)

General Strategy
1. Read the question stem to determine the question type
2. Read the passage (argument) and focus on the required
information for that question type
3. Check all answer choices

500+ videos

800+ practice questions

step-by-step learning guide

All you need. All for free

Weaken the Argument Questions

(watch the entire video here)

Question stem examples


Which of the following, if true, most seriously weakens the
argument above?
Which of the following, if true, would cast the most serious
doubt on the researchers conclusion?
Which of the following, if true, most effectively challenges
the conclusion that releasing 100 Dragonfish into Chilliwack
Lake will allow the Gigafish in Chilliwack Lake to survive?
Which of the following, if established, most seriously calls into
question the argument above?
Which of the following, if true, most seriously undermines the
spokespersons position?
Which of the new premises hurts the argument the most?

500+ videos

800+ practice questions

step-by-step learning guide

All you need. All for free

Weaken the Argument Questions

(watch the entire video here)

General Strategy
1. Read the question stem to determine the question type
2. Read the passage (argument) and focus on the required
information for that question type
3. Check all answer choices

500+ videos

800+ practice questions

step-by-step learning guide

All you need. All for free

Weaken the Argument Questions

(watch the entire video here)

Weaken the Argument Strategy


1. Identify and summarize the conclusion
2. Identify and summarize the premises
3. Identify any assumptions
4. Check each answer choice while
repeating conclusion

Does this weaken


the conclusion
that...?

5. Check all answer choices

500+ videos

800+ practice questions

step-by-step learning guide

All you need. All for free

Weaken the Argument Questions

(watch the entire video here)

Tips
1. Look for common argument types (cause and effect,
statistical, analogy)
2. Common ways to weaken an argument
- Undermining an unstated assumption
- Adding a new premise that hurts the conclusion
3. Weaken does not necessarily mean destroy
4. Beware of answer choices that strengthen the argument
5. Do not try to disprove a premise
6. Goal: Weaken the extent to which the conclusion follows
from the premises

500+ videos

800+ practice questions

step-by-step learning guide

All you need. All for free

Weaken the Argument Questions

(watch the entire video here)

Weaken the Argument Strategy


1. Identify and summarize the conclusion
2. Identify and summarize the premises
3. Identify any assumptions
4. Check each answer choice while
repeating conclusion

Does this weaken


the conclusion
that...?

Example conclusion: Eating carrots improves ones eyesight


5. Check all answer choices

500+ videos

800+ practice questions

step-by-step learning guide

All you need. All for free

Practice Question
A program instituted in a particular state allows parents to prepay their
children's future college tuition at current rates. The program then pays
the tuition annually for the child at any of the state's public colleges in
which the child enrolls. Parents should participate in the program as a
means of decreasing the cost of their children's college education.
Which of the following, if true, is the most appropriate reason for
parents not to participate in the program?
A. The parents are unsure about which public college in
the state the child will attend.
B. The amount of money accumulated by putting the
prepayment funds in an interest-bearing account
today will be greater than the total cost of tuition for
any of the public colleges when the child enrolls.
C. The annual cost of tuition at the state's public
colleges is expected to increase at a faster rate than
the annual increase in the cost of living.
D. Some of the state's public colleges are contemplating
large increases in tuition next year.
E. The prepayment plan would not cover the cost of
room and board at any of the state's public colleges.

This is a question from the


GMAT mini-test. GMAT
questions are the property of the
Graduate Management Admission
Council and are used here with
their permission.

Practice Question

(watch the entire video here)

A program instituted in a particular state allows parents to prepay their


children's future college tuition at current rates. The program then pays
the tuition annually for the child at any of the state's public colleges in
which the child enrolls. Parents should participate in the program as a
means of decreasing the cost of their children's college education.
Which of the following, if true, is the most appropriate reason for
parents not to participate in the program?
A. The parents are unsure about which public college in

the state the child will attend.

P: Program: prepay
tuition @
current rates

P: Program pays
B. The amount of money accumulated by putting the
tuition later
prepayment funds in an interest-bearing account
today will be greater than the total cost of tuition for C: Program is
cheaper
any of the public colleges when the child enrolls.

C. The annual cost of tuition at the state's public

colleges is expected to increase at a faster rate than


the annual increase in the cost of living.

A: Todays cost less


than future cost

D. Some of the state's public colleges are contemplating


large increases in tuition next year.

E. The prepayment plan would not cover the cost of

room and board at any of the state's public colleges.

500+ videos

800+ practice questions

step-by-step learning guide

All you need. All for free

Practice Question
To attract members away from other fitness clubs in the city, FitnessTown
recently launched a new marketing campaign in which each new member
receives a free locker rental, free fitness consultations, and unlimited use
of the tanning beds. FitnessTowns marketing team claims that the
campaign has been a huge success, since its membership has already
increased 20 percent in the first two weeks of the campaign.

Which of the following, if true, would best help to refute the claim of
FitnessTowns marketing team?
A. Almost all of FitnessTowns new members had never
visited a fitness club before they were enticed by
FitnessTowns new campaign.
B. Most of FitnessTowns new members do not intend to
use the free services offered in the campaign.
C. FitnessTowns investors hope to increase membership
by 50 percent within the first month of the campaign.
D. FitnessTowns new marketing campaign is identical
to its previous marketing campaigns.
E. Studies show that people typically cancel their fitness
club memberships within the first year.

500+ videos

800+ practice questions

step-by-step learning guide

All you need. All for free

Practice Question

(watch the entire video here)

To attract members away from other fitness clubs in the city, FitnessTown
recently launched a new marketing campaign in which each new member
receives a free locker rental, free fitness consultations, and unlimited use
of the tanning beds. FitnessTowns marketing team claims that the
campaign has been a huge success, since its membership has already
increased 20 percent in the first two weeks of the campaign.

Which of the following, if true, would best help to refute the claim of
FitnessTowns marketing team?
P: Campaign =
A. Almost all of FitnessTowns new members had never
visited a fitness club before they were enticed by
FitnessTowns new campaign.
B. Most of FitnessTowns new members do not intend to

use the free services offered in the campaign.

C. FitnessTowns investors hope to increase membership

locker, consult.
& tanning

P: Membership up
20% in 2 wks
C: FitnessTown
stole others
members

by 50 percent within the first month of the campaign. A: membership

D. FitnessTowns new marketing campaign is identical

to its previous marketing campaigns.

increase means
stolen
customers.

E. Studies show that people typically cancel their fitness

club memberships within the first year.

500+ videos

800+ practice questions

step-by-step learning guide

All you need. All for free

Strengthen the Argument Questions

500+ videos

800+ practice questions

(watch the entire video here)

step-by-step learning guide

All you need. All for free

Strengthen the Argument Questions


Premise
Premise
Premise
Assumption
+ Assumption

Conclusion

(A) New
(B) New
(C) New
(D) New
(E) New

(watch the entire video here)

Premise
Premise
Premise
Premise
Premise

Goal: Find the answer choice that, when added to


the argument, strengthens the conclusion the most.

500+ videos

800+ practice questions

step-by-step learning guide

All you need. All for free

Strengthen the Argument Questions

(watch the entire video here)

Question stem examples


Which of the following, if true, would provide the most
support for the conclusion of the argument above?
Which of the following statements, if true, most strengthens
the authors argument?
Which of the following, if true, provides the best indication
that Sukis decision was logically sound?
Which of the following, if true, most strongly supports the
recommendation made by the argument?
Which of the new premises improves the
argument the most?

500+ videos

800+ practice questions

step-by-step learning guide

All you need. All for free

Strengthen the Argument Questions

(watch the entire video here)

Strengthen the Argument Strategy


1. Identify and summarize the conclusion
2. Identify and summarize the premises
3. Identify any assumptions
4. Check each answer choice while
repeating conclusion

Does this strengthen


the conclusion
that...?

Example conclusion: Antonio makes the worlds best spaghetti


5. Check all answer choices

500+ videos

800+ practice questions

step-by-step learning guide

All you need. All for free

Strengthen the Argument Questions

(watch the entire video here)

Tips
Look for common argument types (cause and effect,
statistical, analogy)
Common ways to strengthen an argument
- Stating a previously-unstated assumption
- Supporting or elaborating on an existing premise
- Adding a new supporting premise
The goal is not to create a perfect argument
Beware of answer choices that weaken the argument
Watch out for answer choices that support a premise but
not the conclusion
Avram is world-class sprinter
Avram is great chess player
+ Bonnie is world-class sprinter
Bonnie must be great chess player

500+ videos

800+ practice questions

(A) Bonnie holds 100m world record


(B) .
(C) .
(D) .
(E) .

step-by-step learning guide

All you need. All for free

Strengthen the Argument Questions

(watch the entire video here)

Strengthen the Argument Strategy


1. Identify and summarize the conclusion
2. Identify and summarize the premises
3. Identify any assumptions
4. Check each answer choice while
repeating conclusion

Does this strengthen


the conclusion
that...?

5. Check all answer choices

500+ videos

800+ practice questions

step-by-step learning guide

All you need. All for free

Practice Question
In the 12 years since the pretzel desalinization plant was built on the
Polahoochi River, the salinity of the river has increased from 180 S/cm to 911
S/cm in the 10-mile stretch of river downstream from the plant. During the
same 12-year period, researchers have observed a sharp decline in the
average number of eggs laid by individual polafish, a species native to the
Polahoochi River. Given this information, Dr. Kim hypothesizes that high
salinity levels adversely affect the reproductive organs of the polafish.

Which of the following, if true, would most strengthen Dr. Kims hypothesis?
A. In the same state, many populations of polafish that
are not downstream of pretzel desalinization plants
have experienced a reduction in egg production.
B. Prior to the construction of the pretzel desalinization
plant, the salinity of the 10-mile stretch of river
downstream never exceeded 180 S/cm.
C. Other species of fish in the same 10-mile stretch have
experienced a sharp reduction in egg production.
D. In the past 12 years, the salinity of the tributaries
flowing into the 10-mile stretch of river downstream
from the plant has remained below 180 S/cm, and
the polafish living in these tributaries have not
experienced any decline in egg production.
E. In other states, fish downstream from pretzel desalinization
plants have experienced declines in egg production.

500+ videos

800+ practice questions

step-by-step learning guide

All you need. All for free

Practice Question

(watch the entire video here)

In the 12 years since the pretzel desalinization plant was built on the
Polahoochi River, the salinity of the river has increased from 180 S/cm to 911
S/cm in the 10-mile stretch of river downstream from the plant. During the
same 12-year period, researchers have observed a sharp decline in the
average number of eggs laid by individual polafish, a species native to the
Polahoochi River. Given this information, Dr. Kim hypothesizes that high
salinity levels adversely affect the reproductive organs of the polafish.

Which of the following, if true, would most strengthen Dr. Kims hypothesis?
A. In the same state, many populations of polafish that

are not downstream of pretzel desalinization plants


have experienced a reduction in egg production.

B. Prior to the construction of the pretzel desalinization

plant, the salinity of the 10-mile stretch of river


downstream never exceeded 180 S/cm.

C. Other species of fish in the same 10-mile stretch have

experienced a sharp reduction in egg production.

D. In the past 12 years, the salinity of the tributaries


flowing into the 10-mile stretch of river downstream
from the plant has remained below 180 S/cm, and
the polafish living in these tributaries have not
experienced any decline in egg production.

P: plant blt 12 yrs ago


P: downstream: salt
P: downstream:
polafish eggs
C: salinity lowers
polafish egg #s
A: salinity is only
possible cause of
egg

E. In other states, fish downstream from pretzel desalinization

plants have experienced declines in egg production.

500+ videos

800+ practice questions

step-by-step learning guide

All you need. All for free

Assumption Questions

500+ videos

800+ practice questions

(watch the entire video here)

step-by-step learning guide

All you need. All for free

Assumption Questions
Premise
Premise
Premise
Assumption
+ Assumption

Conclusion

(watch the entire video here)

(A) Assumption
(B) Assumption
(C) Assumption
(D) Assumption
(E) Assumption

Goal: Find a necessary assumption

500+ videos

800+ practice questions

step-by-step learning guide

All you need. All for free

Assumption Questions

(watch the entire video here)

Question stem examples


Which of the following is an assumption on which the
argument depends?
The scientists argument depends on the assumption that
The conclusion above follows logically if which of the
following is assumed?

500+ videos

800+ practice questions

step-by-step learning guide

All you need. All for free

Assumption Questions

(watch the entire video here)

Assumption Question Strategy


1. Identify and summarize the conclusion
2. Identify and summarize the premises
3. Identify any assumptions
4. Look for one of your assumptions among
the answer choices
5. Check each answer choice against the
conclusion

Is this assumption
necessary to draw the
conclusion that...?

Example conclusion: Hotdogs are bad for ones health

500+ videos

800+ practice questions

step-by-step learning guide

All you need. All for free

Assumption Questions

(watch the entire video here)

Strengthen the Argument Questions


Want information that helps the conclusion

Assumption Questions
Want information that is necessary for the conclusion

500+ videos

800+ practice questions

step-by-step learning guide

All you need. All for free

Assumption Questions

(watch the entire video here)

Negation Technique
Basis:

1) An assumption is absolutely necessary for


a conclusion to follow from the premises
2) Negating a necessary assumption will destroy
the argument

Juan has been practicing tennis 3 hours each day for the past 2
years. Therefore, Juan will win the city championship next month.
P:
P:
A:
+ A:

J practicing 3hr/day for 2 yrs


J dies before championship
Nothing stops championship
J is eligible to play

C: J will win championship

A: J lives until championship

A: It is not the case that J lives until championship


P: J dies before championship

500+ videos

800+ practice questions

step-by-step learning guide

All you need. All for free

Assumption Questions

(watch the entire video here)

Assumption Question Strategy


1. Identify and summarize the conclusion
2. Identify and summarize the premises
3. Identify any assumptions
4. Look for one of your assumptions among
the answer choices
5. Check each answer choice against the
conclusion
6. Apply Negation Technique:

Is this assumption
necessary to draw the
conclusion that...?

Negate each answer choice and insert it into the argument.


The negated answer choice that destroys the argument is
the correct answer
7. Check all answer choices

500+ videos

800+ practice questions

step-by-step learning guide

All you need. All for free

Assumption Questions

(watch the entire video here)

Tips

Look for common argument types (cause and effect,


statistical, analogy)

Look for shifts in language between premises and


conclusion

Remember that arguments can have any number of


assumptions

Juan has been practicing tennis 3 hours each day for the past 2
years. Therefore, Juan will win the city championship next month.
P:
A:
A:
+ A:

J practicing 3hr/day for 2 mths


J lives until championship
Nothing stops championship
J is eligible to play

C: J will win championship

500+ videos

800+ practice questions

step-by-step learning guide

All you need. All for free

Practice Question
A leading investment firm has issued new rules that prevent its portfolio
managers from short selling stocks for their corporate clients. One
portfolio manager has concluded that this restriction prevents portfolio
managers from providing above-average investment returns for their
corporate clients.
Which of the following is an assumption that would allow the portfolio
managers conclusion to be properly drawn?
A. Investment firms should not restrict the
ways in which portfolio managers
manage their clients funds.
B. Portfolio managers can provide aboveaverage returns for their corporate
clients only by short selling stocks.
C. Short selling is a technique used
primarily for corporate clients.
D. Portfolio managers often used short
selling techniques to provide aboveaverage returns for corporate clients.
E. Before the investment firm issued the
new rules, portfolio managers were not
permitted to short sell stocks.
500+ videos

800+ practice questions

step-by-step learning guide

All you need. All for free

Practice Question

(watch the entire video here)

A leading investment firm has issued new rules that prevent its portfolio
managers from short selling stocks for their corporate clients. One
portfolio manager has concluded that this restriction prevents portfolio
managers from providing above-average investment returns for their
corporate clients.
Which of the following is an assumption that would allow the portfolio
managers conclusion to be properly drawn?
A. Investment firms should not restrict the

ways in which portfolio managers


manage their clients funds.

B. Portfolio managers can provide aboveaverage returns for their corporate


clients only by short selling stocks.

P: New rules prevent SS


C: Rules stop big $ for CC
A: SS necessary to make big $

C. Short selling is a technique used

primarily for corporate clients.

D. Portfolio managers often used short

selling techniques to provide aboveaverage returns for corporate clients.

E. Before the investment firm issued the

new rules, portfolio managers were not


permitted to short sell stocks.

500+ videos

800+ practice questions

step-by-step learning guide

All you need. All for free

Conclusion/Inference Questions

500+ videos

800+ practice questions

(watch the entire video here)

step-by-step learning guide

All you need. All for free

Conclusion/Inference Questions

Premise
Premise
Premise
?

(watch the entire video here)

(A) Conclusion
(B) Conclusion
(C) Conclusion
(D) Conclusion
(E) Conclusion

Goal: Find conclusion that logically follows

500+ videos

800+ practice questions

step-by-step learning guide

All you need. All for free

Conclusion/Inference Questions

(watch the entire video here)

Question stem examples


The statements above, if true, most strongly support which of
the following conclusions?
If the statements above are true, which of the following must
also be true on the basis of them?
Which of the following hypotheses receives the strongest
support from the given information?
Which of the following can be logically inferred based on the
statements above?
Identify something that must follow from the premises

Inference question = Conclusion question

500+ videos

800+ practice questions

step-by-step learning guide

All you need. All for free

Conclusion/Inference Questions

(watch the entire video here)

Typical Conclusion (in most GMAT questions)


Conclusion is partially supported
For the past 3 days, all of Floridas orange farms have
experienced freezing temperatures. Therefore, the number
of oranges harvested this year will be less than expected.
Conclusion in a Conclusion question

Conclusion is guaranteed
For the past 3 days, the temperature at every Florida orange
farm has not exceeded -5 degrees Celsius.
The statement above, if true, most strongly supports which of
the following conclusions?
number of oranges harvested this year will be less
(A) The
than expected.

500+ videos

800+ practice questions

step-by-step learning guide

All you need. All for free

Conclusion/Inference Questions

(watch the entire video here)

Typical Conclusion (in most GMAT questions)


Conclusion is partially supported
For the past 3 days, all of Floridas orange farms have
experienced freezing temperatures. Therefore, the number
of oranges harvested this year will be less than expected.
Conclusion in a Conclusion question

Conclusion is guaranteed
For the past 3 days, the temperature at every Florida orange
farm has not exceeded -5 degrees Celsius.
The statement above, if true, most strongly supports which of
the following conclusions?
orange harvest at some Florida farms will be less
(A) The
than expected.

500+ videos

800+ practice questions

step-by-step learning guide

All you need. All for free

Conclusion/Inference Questions

(watch the entire video here)

Typical Conclusion (in most GMAT questions)


Conclusion is partially supported
For the past 3 days, all of Floridas orange farms have
experienced freezing temperatures. Therefore, the number
of oranges harvested this year will be less than expected.
Conclusion in a Conclusion question

Conclusion is guaranteed
For the past 3 days, the temperature at every Florida orange
farm has not exceeded -5 degrees Celsius.
The statement above, if true, most strongly supports which of
the following conclusions?
least one orange will be damaged from the freezing
(A) At
temperatures.

500+ videos

800+ practice questions

step-by-step learning guide

All you need. All for free

Conclusion/Inference Questions

(watch the entire video here)

Typical Conclusion (in most GMAT questions)


Conclusion is partially supported
For the past 3 days, all of Floridas orange farms have
experienced freezing temperatures. Therefore, the number
of oranges harvested this year will be less than expected.
Conclusion in a Conclusion question

Conclusion is guaranteed
For the past 3 days, the temperature at every Florida orange
farm has not exceeded -5 degrees Celsius.
The statement above, if true, most strongly supports which of
the following conclusions?
the past 3 days, not one Florida orange farm has
(A) For
experienced temperatures above -5 degrees Celsius.

500+ videos

800+ practice questions

step-by-step learning guide

All you need. All for free

Conclusion/Inference Questions

(watch the entire video here)

Typical Conclusion (in most GMAT questions)


Conclusion is partially supported
For the past 3 days, all of Floridas orange farms have
experienced freezing temperatures. Therefore, the number
of oranges harvested this year will be less than expected.
Conclusion in a Conclusion question

Conclusion is guaranteed
For the past 3 days, the temperature at every Florida orange
farm has not exceeded -5 degrees Celsius.
The statement above, if true, most strongly supports which of
the following conclusions?
the past 3 days, at least one Florida orange farm has
(A) During
experienced temperatures lower than 40 degrees Celsius.

500+ videos

800+ practice questions

step-by-step learning guide

All you need. All for free

Conclusion/Inference Questions

(watch the entire video here)

Conclusion Question Strategy


1. Identify and summarize the premises
2. Draw a conclusion that must follow
3. Look for your conclusion among the
answer choices

Must it be true
that...?

4. Aggressively eliminate incorrect answers

5. Apply a version of the Negation Technique:


The negated conclusion that contradicts the premises the
most is probably the correct answer.
6. Check all answer choices

500+ videos

800+ practice questions

step-by-step learning guide

All you need. All for free

Conclusion/Inference Questions

(watch the entire video here)

Tips
1. Do not stray too far from the premises
2. Look for a rewording of a premise
Example
P: Kyle enjoys chocolate
P: Rome is the capital of Italy
C:

What conclusion can be drawn


using every premise?

500+ videos

800+ practice questions

step-by-step learning guide

All you need. All for free

Conclusion/Inference Questions

(watch the entire video here)

Tips
1. Do not stray too far from the premises
2. Look for a rewording of a premise
Example
P: Kyle enjoys chocolate
P: Rome is the capital of Italy
C: The capital of Italy is Rome

500+ videos

800+ practice questions

step-by-step learning guide

All you need. All for free

Conclusion/Inference Questions

(watch the entire video here)

Tips
1. Do not stray too far from the premises
2. Look for a rewording of a premise
3. Conclusions need not involve every premise
4. Do not inject assumptions into the argument
5. Beware of answer choices that introduce new ideas/words
6. Beware of answer choices where the strength of the language
does not match the strength of the language in the premises

500+ videos

800+ practice questions

step-by-step learning guide

All you need. All for free

Practice Question
While studying Emperor Penguins in Antarctica during the coldest 3 months
of the year, researchers observed that several of the penguins died. The
researchers also discovered that the death rate among the larger penguins
was greater than the death rate among the smaller penguins. However, the
researchers conclusion that size was a determinant in these deaths is probably
mistaken, since smaller penguins are typically younger than larger ones.
If the above statements are true, which of the following can be inferred?
A. Among Emperor Penguins of the same age,
the smaller penguins have a greater survival
rate over a 3-month period than the larger
penguins do.
B. Among the Emperor Penguins that survived
the 3-month period, there is probably no
relationship between age and size.
C. Among Emperor Penguins of the same age, a
difference in size may not indicate a difference
in chances of survival over a 3-month period.

D. Temperature does not play a role in the death


rate among Emperor Penguins over a 3-month
period.
E. Among Emperor Penguins of the same size,
age is a determinant in the survival rate over
a 3-month period.

500+ videos

800+ practice questions

step-by-step learning guide

All you need. All for free

Practice Question

(watch the entire video here)

While studying Emperor Penguins in Antarctica during the coldest 3 months


of the year, researchers observed that several of the penguins died. The
researchers also discovered that the death rate among the larger penguins
was greater than the death rate among the smaller penguins. However, the
researchers conclusion that size was a determinant in these deaths is probably
mistaken, since smaller penguins are typically younger than larger ones.
If the above statements are true, which of the following can be inferred?
A. Among Emperor Penguins of the same age,

the smaller penguins have a greater survival P: Ps died during 3 coldest mnths
P: Larger Ps more likely to die
rate over a 3-month period than the larger
penguins do.
P: Resrchrs conclude size a factor
B. Among the Emperor Penguins that survived
P: Size probably not a factor

the 3-month period, there is probably no


P: Smaller Ps typically younger
relationship between age and size.
C. Among Emperor Penguins of the same age, a
difference in size may not indicate a difference
in chances of survival over a 3-month period.

D. Temperature does not play a role in the death

rate among Emperor Penguins over a 3-month


period.

E. Among Emperor Penguins of the same size,

age is a determinant in the survival rate over


a 3-month period.

500+ videos

800+ practice questions

step-by-step learning guide

All you need. All for free

Structure Questions

500+ videos

800+ practice questions

(watch the entire video here)

step-by-step learning guide

All you need. All for free

Structure Questions

(watch the entire video here)

Test your understanding of the argumentative strategies


employed in an argument
3 types of Structure Questions:
- Method of Reasoning
- Boldface
- Parallel Argument

500+ videos

800+ practice questions

step-by-step learning guide

All you need. All for free

Structure Questions

(watch the entire video here)

Method of Reasoning questions


Premise
Premise
Premise
Assumption
+ Assumption
Conclusion

(A) Description
(B) Description
(C) Description
(D) Description
(E) Description

of
of
of
of
of

argument
argument
argument
argument
argument

Goal: Find the best description of the authors


argumentative strategy.

500+ videos

800+ practice questions

step-by-step learning guide

All you need. All for free

Structure Questions

(watch the entire video here)

Question stem examples for Method of Reasoning questions


The authors point is made by which method of reasoning?
Which of the following strategies does Dr. Kwan use to
defend his position?
In the passage, the author develops the argument by ___
The reporter challenges the spokespersons position by
doing which of the following?
Explain how the author presents his/her argument

500+ videos

800+ practice questions

step-by-step learning guide

All you need. All for free

Structure Questions

(watch the entire video here)

Examples of answer choices for Method of Reasoning questions


The argument arrives at its conclusion by demonstrating
the inherent problems with alternative conclusions.

The author offers a new definition of a term that is central


to an opposing argument.
The argument employs circular reasoning by assuming
that which it is trying to prove.

The answer choices are typically generic

500+ videos

800+ practice questions

step-by-step learning guide

All you need. All for free

Structure Questions

(watch the entire video here)

Strategy for tackling Method of Reasoning questions


1. Read the passage
2. For each sentence, ask, What role does this play in
the argument?
3. Identify and summarize the conclusion and premises
4. Use generic language to describe the method of
reasoning to yourself

5. Look for your description among the answer choices


6. Check all answer choices

500+ videos

800+ practice questions

step-by-step learning guide

All you need. All for free

Structure Questions

(watch the entire video here)

Boldface questions
Premise
Premise
Premise
Assumption
+ Assumption
Conclusion

(A) Role
(B) Role
(C) Role
(D) Role
(E) Role

played
played
played
played
played

by
by
by
by
by

boldfaced
boldfaced
boldfaced
boldfaced
boldfaced

portion(s)
portion(s)
portion(s)
portion(s)
portion(s)

Goal: Find the best description of the role(s) played

500+ videos

800+ practice questions

step-by-step learning guide

All you need. All for free

Structure Questions

(watch the entire video here)

Question stem examples for Boldface questions


In the above argument, the portion in boldface plays
which of the following roles?
In the researchers argument, the two portions in
boldface play which of the following roles?

The passage contains bolded text


Researcher: Two years ago, a wolf pack was
relocated to Bilford Island. Although the local
rabbit population has decreased drastically
since the relocation, the wolves are not to
blame for this decrease. Our study shows
that the unprecedented number of recent
rabbit deaths is due to the myxoma virus.
In the above argument, the portion in boldface
plays which of the following roles?

500+ videos

800+ practice questions

step-by-step learning guide

All you need. All for free

Structure Questions

(watch the entire video here)

Strategy for tackling Boldface questions


1. Read the passage
2. For each boldfaced portion, ask, What role does this
play in the argument?
3. Identify and summarize the conclusion and premises
4. Use generic language to describe the roles played by the
boldfaced portion(s)

5. Find the answer choice that most closely matches yours


6. Check all answer choices

500+ videos

800+ practice questions

step-by-step learning guide

All you need. All for free

Structure Questions

(watch the entire video here)

Tips for Boldface questions


1. Look for common roles:
-

Concluding
Summarizing
Contradicting
Providing supporting evidence
Providing an example
Providing a counterexample
Generalizing

2. Consider how the second bolded part is related to


first bolded part

3. Beware of answer choices that are half right and half


wrong

500+ videos

800+ practice questions

step-by-step learning guide

All you need. All for free

Structure Questions

(watch the entire video here)

Parallel Argument questions


Premise
Premise
Premise
Assumption
+ Assumption
Conclusion

(A) Complete
(B) Complete
(C) Complete
(D) Complete
(E) Complete

argument
argument
argument
argument
argument

Goal: Find the argument that employs the


most similar argumentative strategy.

500+ videos

800+ practice questions

step-by-step learning guide

All you need. All for free

Structure Questions

(watch the entire video here)

Question stem examples for Parallel Argument questions


Which of the following arguments exhibits a pattern of
reasoning most similar to the pattern of reasoning
exhibited in the argument above?

Which of the following is most like the argument above


in its logical structure?
The pattern of reasoning displayed above is most
closely paralleled in which of the following?

Find the argument most like the original

500+ videos

800+ practice questions

step-by-step learning guide

All you need. All for free

Structure Questions

(watch the entire video here)

Strategy for tackling Parallel Argument questions


1. Identify and summarize the conclusion and premises
2. Use generic language to describe the method of reasoning
to yourself before checking the answer choices
3. Look for an argument with same structure
4. Check all answer choices

500+ videos

800+ practice questions

step-by-step learning guide

All you need. All for free

Structure Questions

(watch the entire video here)

Tips for Parallel Argument questions


1. Beware of answer choices with same subject matter
2. Questions are time-consuming

500+ videos

800+ practice questions

check your time

step-by-step learning guide

All you need. All for free

Structure Questions

(watch the entire video here)

Test your understanding of the argumentative strategies


employed in an argument
3 types of Structure Questions:
- Method of Reasoning
- Boldfaced
- Parallel Argument

500+ videos

800+ practice questions

step-by-step learning guide

All you need. All for free

Practice Question
ABC Widgets plan to decrease employee salaries by 15% makes sense.
Granted, the reduced salaries will place a financial burden on many
employees. But, if the company does not decrease employee salaries, it
will not have enough money to stay in business, and everyone at the
company will lose his or her job.
The passage employs which of the following argumentative strategies?
A. It explains why the alternative course
of action would not be subject to the
objections raised against the
proposed course of action.
B. It describes an ideal situation by way
of a situation that is less than ideal.
C. It indirectly arrives at its conclusion
by providing grounds to reject an
alternative approach.
D. It extrapolates the perceived outcome of
the proposed plan in order to criticize the
alternative course of action.

E. It employs circular reasoning by assuming


that which it is trying to prove.
500+ videos

800+ practice questions

step-by-step learning guide

All you need. All for free

Practice Question

(watch the entire video here)

ABC Widgets plan to decrease employee salaries by 15% makes sense.


Granted, the reduced salaries will place a financial burden on many
employees. But, if the company does not decrease employee salaries, it
will not have enough money to stay in business, and everyone at the
company will lose his or her job.
The passage employs which of the following argumentative strategies?
P: 15% cut will hurt many

A. It explains why the alternative course

P: No cut no jobs

of action would not be subject to the


objections raised against the
proposed course of action.

B. It describes an ideal situation by way

of a situation that is less than ideal.

C: 15% cut makes sense

Not doing X is much worse than


doing X. So, we should do X.

C. It indirectly arrives at its conclusion


by providing grounds to reject an
alternative approach.

D. It extrapolates the perceived outcome of

the proposed plan in order to criticize the


alternative course of action.

E. It employs circular reasoning by assuming

that which it is trying to prove.

500+ videos

800+ practice questions

step-by-step learning guide

All you need. All for free

Practice Question
Fred: Columbus was a great explorer because he held on to his conviction that
the Earth was round in the face of overwhelming opposition.
Stan: The mark of a great explorer is bravery not the adherence to a
conviction. Besides, the concept of a round Earth was widely accepted
in the fifteenth century, when Columbus was looking for someone to
fund his voyage.
In Stans response, the two boldface portions play which of the following
roles?
A. The first presents an alternative
conclusion; the second provides evidence
in support of that conclusion.

B. The first rejects the criterion on which


Freds argument is based; the second
disputes a specific claim.
C. The first presents an alternative criterion;
the second describes a premise on which
Stans conclusion relies.

D. The first elaborates on Freds criterion;


the second presents a premise on which
Stans conclusion relies.
E. The first reveals a contradiction in Freds
argument; the second resolves that
contradiction.

500+ videos

800+ practice questions

step-by-step learning guide

All you need. All for free

Practice Question

(watch the entire video here)

Fred: Columbus was a great explorer because he held on to his conviction that
the Earth was round in the face of overwhelming opposition.
Stan: The mark of a great explorer is bravery not the adherence to a
conviction. Besides, the concept of a round Earth was widely accepted
in the fifteenth century, when Columbus was looking for someone to
fund his voyage.
In Stans response, the two boldface portions play which of the following
roles?
Fred
A. The first presents an alternative

P: Conviction despite opposition


conclusion; the second provides evidence
C: Columbus great explorer
in support of that conclusion.

B. The first rejects the criterion on which


Freds argument is based; the second
disputes a specific claim.
C. The first presents an alternative criterion;

the second describes a premise on which


Stans conclusion relies.

D. The first elaborates on Freds criterion;

Stan
1st: Great = bravery not conviction
2nd: Round earth known
Stan
1st: Refutes standard
2nd: Calls statement false

the second presents a premise on which


Stans conclusion relies.

E. The first reveals a contradiction in Freds

argument; the second resolves that


contradiction.

500+ videos

800+ practice questions

step-by-step learning guide

All you need. All for free

Practice Question
In Townville, most smokers play tennis, and most nonsmokers do not play tennis.
Therefore, in Townville, most tennis players smoke.
Which of the following exhibits a pattern of flawed reasoning most similar to that in
the argument above?
A. In Townville, most Lions Club members were born in
Townville, and most of the residents who are not Lions
Club members were not born in Townville. Therefore,
most of the residents who were born in Townville are
Lions Club members.
B. In Townville, most of the people who live west of Main
Street own a GPS, and most of the people who own a
GPS live east of Main street. Therefore, most of the
people in Townville own a GPS.
C. In Townville, most cat owners own exactly one dog, and
most dog owners own more than one dog. Therefore,
most of the people in Townville who own more than one
dog do not own any cats.
D. In Townville, most tennis players play golf, but not every
golfer plays tennis. Therefore, in Townville, there are
more tennis players than golfers.
E. In Townville, most of the houses are painted red, and
most of the houses have a pool. Therefore, in Townville,
most of the houses are painted red and have a pool.

500+ videos

800+ practice questions

step-by-step learning guide

All you need. All for free

Practice Question

(watch the entire video here)

In Townville, most smokers play tennis, and most nonsmokers do not play tennis.
Therefore, in Townville, most tennis players smoke.
Which of the following exhibits a pattern of flawed reasoning most similar to that in
the argument above?
A. In Townville, most Lions Club members were born in
Townville, and most of the residents who are not Lions
Club members were not born in Townville. Therefore,
most of the residents who were born in Townville are
Lions Club members.

P:

P: ~S

~T

C:

B. In Townville, most of the people who live west of Main

Street own a GPS, and most of the people who own a


GPS live east of Main street. Therefore, most of the
people in Townville own a GPS.

C. In Townville, most cat owners own exactly one dog, and

most dog owners own more than one dog. Therefore,


most of the people in Townville who own more than one
dog do not own any cats.

D. In Townville, most tennis players play golf, but not every

golfer plays tennis. Therefore, in Townville, there are


more tennis players than golfers.

E. In Townville, most of the houses are painted red, and

most of the houses have a pool. Therefore, in Townville,


most of the houses are painted red and have a pool.

500+ videos

800+ practice questions

step-by-step learning guide

All you need. All for free

Flawed Argument Questions

500+ videos

800+ practice questions

(watch the entire video here)

step-by-step learning guide

All you need. All for free

Flawed Argument Questions


Premise
Premise
Premise
Assumption
+ Assumption
Flawed Conclusion

(A) Main
(B) Main
(C) Main
(D) Main
(E) Main

(watch the entire video here)

problem
problem
problem
problem
problem

Goal: Find the arguments primary flaw

500+ videos

800+ practice questions

step-by-step learning guide

All you need. All for free

Flawed Argument Questions

(watch the entire video here)

Question stem examples


Which of the following identifies the most serious logical flaw
in the argument above?
Which one of the following best identifies the error in
reasoning made in the passage?
The argument is vulnerable to criticism on which one of the
following grounds?
The reasoning in the argument is not sound because it fails to
establish that ___

Identify the main problem with the argument

500+ videos

800+ practice questions

step-by-step learning guide

All you need. All for free

Flawed Argument Questions

(watch the entire video here)

Strategy
1. Identify and summarize the conclusion and premises
2. Identify any unstated assumptions
3. Determine the primary flaw
4. Look for your answer among the answer choices
5. Check all answer choices

500+ videos

800+ practice questions

step-by-step learning guide

All you need. All for free

Flawed Argument Questions

(watch the entire video here)

Common Flaws
Confusing causation with correlation

While studying the reading levels of elementary


students, researchers discovered that, on
average, the longer a childs foot, the higher
his/her reading level. Therefore, foot growth
increases ones reading level.

500+ videos

800+ practice questions

step-by-step learning guide

All you need. All for free

Flawed Argument Questions

(watch the entire video here)

Common Flaws
Confusing causation with correlation
Confusing numbers with rates
Last year, 10 people were murdered in
Happyton, and 100 people were murdered
in Killington. Therefore, it is much safer to
live in Happyton than in Killington

500+ videos

800+ practice questions

step-by-step learning guide

All you need. All for free

Flawed Argument Questions

(watch the entire video here)

Common Flaws
Confusing causation with correlation
Confusing numbers with rates
Conclusion mismatch
- Watch out for new words in the conclusion

Last year, hundreds of postal workers in Maltania


suffered back injuries from carrying heavy
packages at the sorting stations. To help reduce
these injuries, the government should introduce a
law that prohibits people from mailing packages
that are over 3 feet in length.

500+ videos

800+ practice questions

step-by-step learning guide

All you need. All for free

Flawed Argument Questions

(watch the entire video here)

Common Flaws
Confusing causation with correlation
Confusing numbers with rates
Conclusion mismatch
- Watch out for new words in the conclusion
Extreme conclusion
In some countries, raising the minimum
wage can help boost the economy. So, if
Maltania raises its minimum wage, its
economy will improve.

500+ videos

800+ practice questions

step-by-step learning guide

All you need. All for free

Flawed Argument Questions

(watch the entire video here)

Common Flaws
Confusing causation with correlation
Confusing numbers with rates
Conclusion mismatch
- Watch out for new words in the conclusion
Extreme conclusion
Mistaking necessary for sufficient
In order to be a great science teacher, one
must have a deep understanding of biology.
Since Terrence has a deep understanding of
biology, he must be a great science teacher.

500+ videos

800+ practice questions

step-by-step learning guide

All you need. All for free

Flawed Argument Questions

(watch the entire video here)

Common Flaws
Confusing causation with correlation
Confusing numbers with rates
Conclusion mismatch
- Watch out for new words in the conclusion
Extreme conclusion
Mistaking necessary for sufficient
Guilty by association
Sharon, Margaret and Rena are tall, Hoopton
High students who play basketball. Since
Maureen is a tall, Hoopton High student, she
must play basketball.

500+ videos

800+ practice questions

step-by-step learning guide

All you need. All for free

Flawed Argument Questions

(watch the entire video here)

Common Flaws
Confusing causation with correlation
Confusing numbers with rates
Conclusion mismatch
- Watch out for new words in the conclusion
Extreme conclusion
Mistaking necessary for sufficient
Guilty by association
Unrepresentative sample
Recently, a questionnaire was given to the
inhabitants of Capton, Maltanias capital city.
83% of respondents said they own one or more
horses. Therefore, we can conclude that most
people in Maltania own one or more horses.

500+ videos

800+ practice questions

step-by-step learning guide

All you need. All for free

Flawed Argument Questions

(watch the entire video here)

Strategy
1. Identify and summarize the conclusion and premises
2. Identify any unstated assumptions
3. Determine the primary flaw
4. Look for your description among the answer choices
5. Check all answer choices

500+ videos

800+ practice questions

step-by-step learning guide

All you need. All for free

Practice Question
Jimmylegs, a disorder of the metatarsals, affects only jabberwockies. A popular
belief contends that the longhaired variety of the jabberwocky is more
predisposed to contract jimmylegs than are other varieties of jabberwocky. This,
however, is clearly a myth, since data collected by the World Jabberwocky
Society (WJS) indicate that, of all the jabberwockies that contract jimmylegs
each year, the majority of the cases involve shorthaired jabberwockies.
Which of the following highlights the most serious flaw in the above argument?
A. The author assumes that the only
varieties of jabberwocky are
longhaired and shorthaired.
B. The author does not consider what
proportion of all cases of jimmylegs
involves longhaired jabberwockies.
C. The author calls into question the
authenticity of the WJS data without
providing any data to the contrary.
D. The author takes no account of the relative
frequency of longhaired jabberwockies
within the jabberwocky population.
E. The author limits the argument to an
unnecessarily restrictive subset of criteria
before drawing a general conclusion.

500+ videos

800+ practice questions

step-by-step learning guide

All you need. All for free

Practice Question

(watch the entire video here)

Jimmylegs, a disorder of the metatarsals, affects only jabberwockies. A popular


belief contends that the longhaired variety of the jabberwocky is more
predisposed to contract jimmylegs than are other varieties of jabberwocky. This,
however, is clearly a myth, since data collected by the World Jabberwocky
Society (WJS) indicate that, of all the jabberwockies that contract jimmylegs
each year, the majority of the cases involve shorthaired jabberwockies.
Which of the following highlights the most serious flaw in the above argument?
A. The author assumes that the only

P: J-legs = jabberwocky disorder


P: Belief: longhair predisposed to j-legs
P: Majority of cases are shorthair
C: Longhair not more predisposed

varieties of jabberwocky are


longhaired and shorthaired.

B. The author does not consider what

proportion of all cases of jimmylegs


involves longhaired jabberwockies.

A: Pop. breakdown supports conclusion

C. The author calls into question the

authenticity of the WJS data without


providing any data to the contrary.

Flaw: We dont know the


population breakdown

D. The author takes no account of the relative


frequency of longhaired jabberwockies
within the jabberwocky population.
E. The author limits the argument to an

unnecessarily restrictive subset of criteria


before drawing a general conclusion.

500+ videos

800+ practice questions

step-by-step learning guide

All you need. All for free

Paradox Questions

500+ videos

800+ practice questions

(watch the entire video here)

step-by-step learning guide

All you need. All for free

Paradox Questions

Premise
Premise
+ Premise
Conclusion

(A) New
(B) New
(C) New
(D) New
(E) New

(watch the entire video here)

Premise
Premise
Premise
Premise
Premise

Goal: Find premise that resolves the paradox

500+ videos

800+ practice questions

step-by-step learning guide

All you need. All for free

Paradox Questions

(watch the entire video here)

Question stem examples


The paradox described above is best resolved by which of
the following?
Which of the following, if true, most helps to resolve the
apparent discrepancy described above?
Which of the following, if true, best explains the
paradoxical outcome of Dr. Doolittle's experiment?
Which one of the following most helps to explain the
apparent contradiction above?

Which one of the following, if true, most helps to explain


the difference in melting points?
Identify something that resolves the
contradictory information

500+ videos

800+ practice questions

step-by-step learning guide

All you need. All for free

Paradox Questions

(watch the entire video here)

Statistics show that the number of smokers in


Maltania has steadily decreased over the past 10
years. However, during the same 10 years, the
total amount of tobacco sold by Maltanian tobacco
farmers has increased.
Which of the following, if true, most helps to
resolve the apparent discrepancy described
above?
Looking for an aha premise
Not testing ability to deconstruct arguments
Several explanations:
- Farmers exporting to other markets
- Big increase in tobacco chewers
- and more . . .

500+ videos

800+ practice questions

step-by-step learning guide

All you need. All for free

Paradox Questions

(watch the entire video here)

Paradox Question strategy


1. Identify the contradictory premises
2. Explain the paradox to yourself
e.g., More tobacco sold despite fewer smokers
3. Check the answer choices while reminding yourself
of the paradox
Does this explain why...?
4. Check all answer choices

500+ videos

800+ practice questions

step-by-step learning guide

All you need. All for free

Paradox Questions

(watch the entire video here)

Tips
Keywords: yet, however, surprisingly, nonetheless, paradoxically
Unable to identify paradox
solution unlikely
reread passage or guess and move on
Beware of answer choices that have opposite effect

Statistics show that the number of smokers in Maltania


has steadily decreased over the past 10 years. However,
during the same 10 years, the total amount of tobacco
sold by Maltanian tobacco farmers has increased.
Which of the following, if true, most helps to resolve the
apparent discrepancy described above?
Maltanian government has introduced stop-smoking
(A) The
programs across the country.

500+ videos

800+ practice questions

step-by-step learning guide

All you need. All for free

Practice Question
Among the mechanics at Joes Garage, the senior mechanics are the most
adept at diagnosing engine problems. However, the itemized bills for engine
repairs made during the past four years suggest otherwise. From the bills, we
see that, on average, the senior mechanics took 29 minutes to correctly
diagnose an engine problem, while the junior mechanics took only 22 minutes.
Which of the following, if true, most helps to resolve the apparent paradox?
A. At Joes Garage, mechanics with more
than three years of experience are
called senior mechanics.
B. Some of the junior mechanics had
experience diagnosing engine problems
before they began working at Joes
Garage.
C. At Joes Garage, the manager typically
assigns to the junior mechanics engine
problems that he expects will be
relatively easy to diagnose.
D. During the past 4 years, most of the
senior mechanics left Joes Garage to
open their own garages.

E. Mechanics who are more adept at


diagnosing engine problems are faster
at completing the required repair.

500+ videos

800+ practice questions

step-by-step learning guide

All you need. All for free

Practice Question

(watch the entire video here)

Among the mechanics at Joes Garage, the senior mechanics are the most
adept at diagnosing engine problems. However, the itemized bills for engine
repairs made during the past four years suggest otherwise. From the bills, we
see that, on average, the senior mechanics took 29 minutes to correctly
diagnose an engine problem, while the junior mechanics took only 22 minutes.
Which of the following, if true, most helps to resolve the apparent paradox?
A. At Joes Garage, mechanics with more

than three years of experience are


called senior mechanics.

B. Some of the junior mechanics had

experience diagnosing engine problems


before they began working at Joes
Garage.

C. At Joes Garage, the manager typically


assigns to the junior mechanics engine
problems that he expects will be
relatively easy to diagnose.

P: Sr. mechs best at diagnosing


P: 29 min for sr. mechs, and
.22 min for jr. mechs
Paradox: Sr. mechanics take
longer despite their superior
diagnostic skills.

D. During the past 4 years, most of the

senior mechanics left Joes Garage to


open their own garages.

E. Mechanics who are more adept at

diagnosing engine problems are faster


at completing the required repair.

500+ videos

800+ practice questions

step-by-step learning guide

All you need. All for free

Evaluate the Conclusion Questions

500+ videos

800+ practice questions

(watch the entire video here)

step-by-step learning guide

All you need. All for free

Evaluate the Conclusion Questions


Premise
Premise
Premise
Assumption
+ Assumption

Conclusion

(watch the entire video here)

(A) Question
(B) Question
(C) Question
(D) Question
(E) Question

Goal: Find the question that, when answered,


best helps to evaluate the conclusion.

500+ videos

800+ practice questions

step-by-step learning guide

All you need. All for free

Evaluate the Conclusion Questions

(watch the entire video here)

Researcher: Two years ago, a wolf pack was relocated to Bilford


Island. Although the local rabbit population has decreased
drastically since the relocation, the wolves are not to blame for
this decrease. Our study shows that the unprecedented number
of recent rabbit deaths is due to the myxoma virus.
Which of the following would be most relevant to investigate in
order to evaluate the researchers conclusion?
Conclusion: Wolves not responsible for population decrease.
A) In what season was the wolf pack relocated to Bilford Island?
B) What proportion of rabbits have died from the myxoma virus?
C) Did the wolf pack introduce the myxoma virus to Bilford Island?
D) . . .

500+ videos

Goal: Find the question that, when answered,


helps evaluate the conclusion

800+ practice questions

step-by-step learning guide

All you need. All for free

Evaluate the Conclusion Questions

(watch the entire video here)

Question stem examples


Knowing which of the following would be most useful in evaluating
the argument?
Which of the following would be most relevant to investigate in
order to evaluate the researchers conclusion?

Clarification of which of the following issues would be most


important to evaluating the spokespersons position?
Identify a question that would help
gauge the strength of the conclusion

500+ videos

800+ practice questions

step-by-step learning guide

All you need. All for free

Evaluate the Conclusion Questions

(watch the entire video here)

Strategy
1. Identify and summarize the conclusion and premises
2. Identify any assumptions
3. Check the answer choices by providing an answer to
each question and relating it to the conclusion
4. Check all answer choices

500+ videos

800+ practice questions

step-by-step learning guide

All you need. All for free

Evaluate the Conclusion Questions

(watch the entire video here)

3. Check the answer choices by providing an answer to


each question and relating it to the conclusion
Goal: Find the question that, when answered,
helps evaluate the conclusion.

500+ videos

800+ practice questions

step-by-step learning guide

All you need. All for free

Evaluate the Conclusion Questions

(watch the entire video here)

3. Check the answer choices by providing an answer to


each question and relating it to the conclusion
Researcher: Two years ago, a wolf pack was relocated to Bilford
Island. Although the local rabbit population has decreased
drastically since the relocation, the wolves are not to blame for
this decrease. Our study shows that the unprecedented number
of recent rabbit deaths is due to the myxoma virus.
Which of the following would be most relevant to investigate to
evaluate the researchers conclusion?
Conclusion: Wolves not responsible for population decrease.

A) In what season was the wolf pack relocated to Bilford Island?

500+ videos

800+ practice questions

step-by-step learning guide

Summer

All you need. All for free

Evaluate the Conclusion Questions

(watch the entire video here)

3. Check the answer choices by providing an answer to


each question and relating it to the conclusion
Researcher: Two years ago, a wolf pack was relocated to Bilford
Island. Although the local rabbit population has decreased
drastically since the relocation, the wolves are not to blame for
this decrease. Our study shows that the unprecedented number
of recent rabbit deaths is due to the myxoma virus.
Which of the following would be most relevant to investigate to
evaluate the researchers conclusion?
Conclusion: Wolves not responsible for population decrease.

A) In what season was the wolf pack relocated to Bilford Island?

500+ videos

800+ practice questions

step-by-step learning guide

Winter

All you need. All for free

Evaluate the Conclusion Questions

(watch the entire video here)

3. Check the answer choices by providing an answer to


each question and relating it to the conclusion
Researcher: Two years ago, a wolf pack was relocated to Bilford
Island. Although the local rabbit population has decreased
drastically since the relocation, the wolves are not to blame for
this decrease. Our study shows that the unprecedented number
of recent rabbit deaths is due to the myxoma virus.
Which of the following would be most relevant to investigate to
evaluate the researchers conclusion?
Conclusion: Wolves not responsible for population decrease.

A) In what season was the wolf pack relocated to Bilford Island?


B) What proportion of rabbits have died from the myxoma virus?

500+ videos

800+ practice questions

step-by-step learning guide

Winter

43%

All you need. All for free

Evaluate the Conclusion Questions

(watch the entire video here)

3. Check the answer choices by providing an answer to


each question and relating it to the conclusion
Researcher: Two years ago, a wolf pack was relocated to Bilford
Island. Although the local rabbit population has decreased
drastically since the relocation, the wolves are not to blame for
this decrease. Our study shows that the unprecedented number
of recent rabbit deaths is due to the myxoma virus.
Which of the following would be most relevant to investigate to
evaluate the researchers conclusion?
Conclusion: Wolves not responsible for population decrease.

A) In what season was the wolf pack relocated to Bilford Island? Winter
B) What proportion of rabbits have died from the myxoma virus? 43%
C) Did the wolf pack introduce the myxoma virus to Bilford Island? Yes

500+ videos

800+ practice questions

step-by-step learning guide

All you need. All for free

Evaluate the Conclusion Questions

(watch the entire video here)

3. Check the answer choices by providing an answer to


each question and relating it to the conclusion
Researcher: Two years ago, a wolf pack was relocated to Bilford
Island. Although the local rabbit population has decreased
drastically since the relocation, the wolves are not to blame for
this decrease. Our study shows that the unprecedented number
of recent rabbit deaths is due to the myxoma virus.
Which of the following would be most relevant to investigate to
evaluate the researchers conclusion?
Conclusion: Wolves not responsible for population decrease.

A) In what season was the wolf pack relocated to Bilford Island? Winter
B) What proportion of rabbits have died from the myxoma virus? 43%
C) Did the wolf pack introduce the myxoma virus to Bilford Island? No

500+ videos

800+ practice questions

step-by-step learning guide

All you need. All for free

Evaluate the Conclusion Questions

(watch the entire video here)

Strategy
1. Identify and summarize the conclusion and premises
2. Identify any assumptions
3. Check the answer choices by providing an answer to
each question and relating it to the conclusion
4. Check all answer choices

500+ videos

800+ practice questions

step-by-step learning guide

All you need. All for free

Practice Question
Professor Gelding: The belief that our planet will experience a wave of extreme
weather events over the next 100 years is nonsense. There is no evidence to
support that the rate of extreme weather events is increasing.
Professor Brink: To see where you are mistaken, you need only examine the
data concerning Category 5 hurricanes in the Gulf of Mexico that made landfall
in the past 100 years. In the first 50 years, there were only 6 such hurricanes,
but in the last 50 years, there were 19 such hurricanes.
Clarification of which of the following issues would be most important to an
evaluation of Professor Brinks conclusion?
A. Does the rate at which Category 5 hurricanes
make landfall affect the rate of other extreme
weather events?
B. Do the statistics concerning Category 5
hurricanes that made landfall represent
extreme weather events in the Gulf of Mexico?
C. During the last 100 years, what percent of
hurricanes made landfall?
D. How many extreme weather events occurred in
the past 100 years?
E. Do the Category 5 hurricanes in the Gulf of
Mexico that made landfall in the past 100 years
represent extreme weather events in general?

500+ videos

800+ practice questions

step-by-step learning guide

All you need. All for free

Practice Question

(watch the entire video here)

Professor Gelding: The belief that our planet will experience a wave of extreme
weather events over the next 100 years is nonsense. There is no evidence to
support that the rate of extreme weather events is increasing.
Professor Brink: To see where you are mistaken, you need only examine the
data concerning Category 5 hurricanes in the Gulf of Mexico that made landfall
in the past 100 years. In the first 50 years, there were only 6 such hurricanes,
but in the last 50 years, there were 19 such hurricanes.
Clarification of which of the following issues would be most important to an
evaluation of Professor Brinks conclusion?
A. Does the rate at which Category 5 hurricanes

P: 1st 50 yrs: 6 hurricanes

make landfall affect the rate of other extreme P: 2 50 yrs: 19 hurricanes


C: EW rate is increasing
weather events?

B. Do the statistics concerning Category 5

hurricanes that made landfall represent


extreme weather events in the Gulf of Mexico?

nd

A: hurricane rate
characterizes extreme
weather rate

C. During the last 100 years, what percent of

hurricanes made landfall?

D. How many extreme weather events occurred in

the past 100 years?

E. Do the Category 5 hurricanes in the Gulf of


Mexico that made landfall in the past 100 years
represent extreme weather events in general?

500+ videos

800+ practice questions

step-by-step learning guide

All you need. All for free

Miscellaneous Tips

500+ videos

800+ practice questions

(watch the entire video here)

step-by-step learning guide

All you need. All for free

Miscellaneous Tips

(watch the entire video here)

Question type frequencies


GMAT words
EXCEPT questions
Being aggressive

500+ videos

800+ practice questions

step-by-step learning guide

All you need. All for free

Miscellaneous Tips

(watch the entire video here)

Question type frequencies


1. Weaken the Argument
2. Strengthen the Argument
3. Assumption
4. Conclusion/Inference
5. Method of Reasoning
6. Flawed Argument
7. Evaluation
8. Paradox

500+ videos

800+ practice questions

step-by-step learning guide

All you need. All for free

Miscellaneous Tips

(watch the entire video here)

Question type frequencies

2. Strengthen the Argument

3. Assumption

4. Conclusion/Inference

1. Weaken the Argument

5. Method of Reasoning
6. Flawed Argument
7. Evaluation
8. Paradox

500+ videos

800+ practice questions

step-by-step learning guide

All you need. All for free

Miscellaneous Tips

(watch the entire video here)

GMAT words
Common usage
Everybody likes ice cream = a lot of people like ice cream
= most people like ice cream
GMAT usage

Everybody likes ice cream = every person likes ice cream

500+ videos

800+ practice questions

step-by-step learning guide

All you need. All for free

Miscellaneous Tips

(watch the entire video here)

GMAT words
Read words in their strongest, most literal sense
all, none, everyone, no one, always, never, each
every, anywhere, nowhere
Some: 1 or more
Some Gigacorp employees are college graduates.
Some countries in Europe are named Italy.
Some of Earths oceans contain salt water
Most: More than 50%
Most of Earths oceans contain salt water

500+ videos

800+ practice questions

step-by-step learning guide

All you need. All for free

Miscellaneous Tips

(watch the entire video here)

EXCEPT questions
Today, Gary did not arrive at the office until 11am.
Therefore Gary will be fired.
Each of the following, if true, weakens the
conclusion above EXCEPT
Does not mean strengthen the conclusion

500+ videos

800+ practice questions

step-by-step learning guide

All you need. All for free

Miscellaneous Tips

(watch the entire video here)

EXCEPT questions
Each of the following people is a former President
of the United States EXCEPT

If the person is a former US president


eliminate it.
(A)
Abraham Lincoln
(B)
George Washington
(C) Bugs Bunny
(D)
Jimmy Carter

(E)
George Bush

500+ videos

800+ practice questions

step-by-step learning guide

All you need. All for free

Miscellaneous Tips

(watch the entire video here)

EXCEPT questions
Today, Gary did not arrive at the office until 11am.
Therefore Gary will be fired.
Each of the following, if true, weakens the
conclusion above EXCEPT
If it weakens the conclusion, eliminate it.
(A)
Work begins at noon
(B)
Gary is self employed
(C) Gary likes ice cream

500+ videos

800+ practice questions

step-by-step learning guide

All you need. All for free

Miscellaneous Tips

(watch the entire video here)

Being aggressive
Critical reasoning questions can be time consuming
Students get lost in the answer choices
Be aggressive!
What if I eliminate all answer choices?
This will not happen often
You will have earned time to recheck answer choices

Look for reasons to reject answer choices

500+ videos

800+ practice questions

step-by-step learning guide

All you need. All for free

Miscellaneous Tips

(watch the entire video here)

Question type frequencies


GMAT words
EXCEPT questions
Being aggressive

500+ videos

800+ practice questions

step-by-step learning guide

All you need. All for free

Practice Question
For decades, countries have measured wheat field productivity using the
Wheat Field Productivity Index (WFPI), which is equal to the average
number of pounds of grain harvested per acre of wheat. In 2005,
Maltanias WFPI was 60 pounds per acre more than Italys WFPI. In 2006,
Maltanias WFPI was 75 pounds per acre more than Italys. Therefore,
Maltanias wheat field productivity must have increased during that period.

Which of the following, if true, provides the strongest support for the
conclusion above?
A. Between 2005 and 2006, the number of
acres of wheat planted in both Maltania
and Italy increased at the same rate.
B. In 2006, Maltania received 20% more
sunshine than usual.
C. Between 2005 and 2006, Italys wheat
field productivity decreased by 10
pounds per acre.
D. In 2006, Italy experienced its worst
drought in decades.
E. In 2006, Maltania began subsidizing all of
its wheat farmers.
500+ videos

800+ practice questions

step-by-step learning guide

All you need. All for free

Practice Question

(watch the entire video here)

For decades, countries have measured wheat field productivity using the
Wheat Field Productivity Index (WFPI), which is equal to the average
number of pounds of grain harvested per acre of wheat. In 2005,
Maltanias WFPI was 60 pounds per acre more than Italys WFPI. In 2006,
Maltanias WFPI was 75 pounds per acre more than Italys. Therefore,
Maltanias wheat field productivity must have increased during that period.

Which of the following, if true, provides the strongest support for the
conclusion above?
A. Between 2005 and 2006, the number of

acres of wheat planted in both Maltania


and Italy increased at the same rate.
B. In 2006, Maltania received 20% more

sunshine than usual.

P: WFPI = ave lbs wheat/acre


P: 2005: Maltanias WFPI 60
more than Italys
P: 2006: Maltanias WFPI 75
more than Italys
C: Maltanias WFPI

C. Between 2005 and 2006, Italys wheat


field productivity decreased by 10
pounds per acre.
D. In 2006, Italy experienced its worst

drought in decades.

E. In 2006, Maltania began subsidizing all of

its wheat farmers.

500+ videos

800+ practice questions

step-by-step learning guide

All you need. All for free

Practice Question
All of Pedros friends say they know someone who is a licensed
circus clown. Pedro does not know any licensed circus clowns, and
he is not unique among his friends in this respect.
If the above statements are true, which of the following must also
be true?
A. Some of Pedros friends are lying.
B. Most licensed circus clowns lie about
their profession.

C. Some of Pedros friends have friends


that Pedro is not friends with.
D. Pedro is a licensed circus clown.
E. All of Pedros friends know the same
licensed circus clown.

500+ videos

800+ practice questions

step-by-step learning guide

All you need. All for free

Practice Question

(watch the entire video here)

All of Pedros friends say they know someone who is a licensed


circus clown. Pedro does not know any licensed circus clowns, and
he is not unique among his friends in this respect.
If the above statements are true, which of the following must also
be true?
P: All friends SAY they know a C

A. Some of Pedros friends are lying.


B. Most licensed circus clowns lie about

their profession.

P: Pedro knows zero Cs


P: Some friends dont know a C

C. Some of Pedros friends have friends

that Pedro is not friends with.

D. Pedro is a licensed circus clown.

E. All of Pedros friends know the same

licensed circus clown.

500+ videos

800+ practice questions

step-by-step learning guide

All you need. All for free

Practice Question
Reporter: The number of foreigners working at US ski resorts was lower
last year than it had been in previous years. Last year, several ski
resorts hired more local residents than they had hired in previous years,
so it is likely that the reduced number of foreign workers at US ski
resorts last year was due to the decreased demand for their services.
Which of the following, if true, most seriously undermines the reporters
argument?
A. All foreigners who received work visas
for last years ski season were able to
find work at US ski resorts.
B. The average wait time to receive a US
work visa has decreased substantially
over the past three years.
C. The number of ski resorts that hired
foreigners last year was the same as in
previous years.
D. Foreigners can work for more than one
ski resort in any given year.
E. Recent changes to US immigration laws
drastically decreased the total number
of work visas issued for last year.
500+ videos

800+ practice questions

step-by-step learning guide

All you need. All for free

Practice Question

(watch the entire video here)

Reporter: The number of foreigners working at US ski resorts was lower


last year than it had been in previous years. Last year, several ski
resorts hired more local residents than they had hired in previous years,
so it is likely that the reduced number of foreign workers at US ski
resorts last year was due to the decreased demand for their services.
Which of the following, if true, most seriously undermines the reporters
argument?

A. All foreigners who received work visas


for last years ski season were able to
find work at US ski resorts.

B. The average wait time to receive a US

work visa has decreased substantially


over the past three years.

C. The number of ski resorts that hired

P: # foreign wrkrs lower last yr


P: More locals hired last yr

C: local employees caused


foreign employees
A: Nothing else caused decrease
A: Fewer foreign workers did not
cause increase in locals hired

foreigners last year was the same as in


previous years.

D. Foreigners can work for more than one


ski resort in any given year.

E. Recent changes to US immigration laws


drastically decreased the total number
of work visas issued for last year.
500+ videos

800+ practice questions

step-by-step learning guide

All you need. All for free

Practice Question
After a plague of locusts attacked every crop in Baker County, Kevin
Kevinson claimed that the severity of damage to his crops was due to
the fish-based fertilizer he applied to his fields before planting.
Which of the following investigations is most likely to yield significant
information that would help evaluate Kevins argument?
A. Comparing the value of Kevins crop damage
to the average value of the crop damage at
farms where fish-based fertilizer was not
applied to the fields before planting
B. Determining the extent to which fish-based
fertilizer increases crop yield
C. Comparing the long-term effects of Kevins
crop damage to the long-term effects of
other crop damage in Baker County
D. Comparing the severity of crop damage at Kevins
farm with the crop damage at agriculturally similar
Baker County farms where fish-based fertilizer was
not applied to fields before planting
E. Determining the percentage of Baker County farms
that applied fish-based fertilizer to their fields
before planting
500+ videos

800+ practice questions

step-by-step learning guide

All you need. All for free

Practice Question

(watch the entire video here)

After a plague of locusts attacked every crop in Baker County, Kevin


Kevinson claimed that the severity of damage to his crops was due to
the fish-based fertilizer he applied to his fields before planting.
Which of the following investigations is most likely to yield significant
information that would help evaluate Kevins argument?
A. Comparing the value of Kevins crop damage

to the average value of the crop damage at


farms where fish-based fertilizer was not
applied to the fields before planting

B. Determining the extent to which fish-based

fertilizer increases crop yield

P: Locusts hit all crops


P: Kevin applied FBF to crops
C: FBF caused severity
A: Nothing else caused the
severity

C. Comparing the long-term effects of Kevins

crop damage to the long-term effects of


other crop damage in Baker County

D. Comparing the severity of crop damage at Kevins


farm with the crop damage at agriculturally similar
Baker County farms where fish-based fertilizer was
not applied to fields before planting
E. Determining the percentage of Baker County farms

that applied fish-based fertilizer to their fields


before planting

500+ videos

800+ practice questions

step-by-step learning guide

All you need. All for free

Practice Question
Ayla is taller than Elan, and Brek is the same height as Diego. Since
Chayna is shorter than Diego, it follows that Ayla is taller than Chayna.
Each of the following, when added to the argument as an additional
premise, makes the argument logically correct EXCEPT:
A. Diego is shorter than Elan.
B. Ayla and Diego are the same height.
C. Brek is taller than Elan.
D. Elan and Chayna are the same height.

E. Brek is shorter than Ayla.

500+ videos

800+ practice questions

step-by-step learning guide

All you need. All for free

Practice Question

(watch the entire video here)

Ayla is taller than Elan, and Brek is the same height as Diego. Since
Chayna is shorter than Diego, it follows that Ayla is taller than Chayna.
Each of the following, when added to the argument as an additional
premise, makes the argument logically correct EXCEPT:
A. Diego is shorter than Elan.

B. Ayla and Diego are the same height.

C. Brek is taller than Elan.


D. Elan and Chayna are the same height.

E. Brek is shorter than Ayla.

500+ videos

800+ practice questions

If it makes the argument


correct, eliminate it.

Premises:

A
B=D
+
E
C

Conclusion:

A
C

step-by-step learning guide

All you need. All for free

Practice Question
To estimate changes in crab populations, biologists monitor what is
known as crabs per trap (CPT). The CPT is the average number of
crabs that commercial crab fishermen catch per crab trap. Since the
current CPT for snow crabs is the same as it was 10 years ago,
biologists conclude that todays snow crab population is approximately
the same as it was 10 years ago.

Which of the following, if true, most seriously weakens the argument?


A. In the past 7 years, the CPT for king crabs
has decreased by 50%.
B. 8 years ago, commercial crab-fishing
boats began using high-tech sonar
equipment that enables them to locate
crabs with much greater accuracy.
C. 9 years ago, a deadly species of sea lice
drove many crab species to near
extinction.
D. In the past 10 years, the number of
commercial crab traps set each year has
remained relatively constant.
E. In the past 10 years, the worldwide price
of snow crabs has more than doubled.
500+ videos

800+ practice questions

step-by-step learning guide

All you need. All for free

Practice Question

(watch the entire video here)

To estimate changes in crab populations, biologists monitor what is


known as crabs per trap (CPT). The CPT is the average number of
crabs that commercial crab fishermen catch per crab trap. Since the
current CPT for snow crabs is the same as it was 10 years ago,
biologists conclude that todays snow crab population is approximately
the same as it was 10 years ago.

Which of the following, if true, most seriously weakens the argument?


A. In the past 7 years, the CPT for king crabs

has decreased by 50%.

B. 8 years ago, commercial crab-fishing


boats began using high-tech sonar
equipment that enables them to locate
crabs with much greater accuracy.

P: CPT = ave # crabs/trap

C. 9 years ago, a deadly species of sea lice

A: CPT reflects pop.

drove many crab species to near


extinction.

P: CPT now = 10 yrs ago


C: SC pop. 10 yrs ago

D. In the past 10 years, the number of

commercial crab traps set each year has


remained relatively constant.

E. In the past 10 years, the worldwide price

of snow crabs has more than doubled.

500+ videos

800+ practice questions

step-by-step learning guide

All you need. All for free

Practice Question
Identical twins have livers that are genetically identical. Scientists recently
discovered that when one twin has hepachrinosis, a debilitating liver disease,
the caudate lobe of the affected twins liver is significantly smaller than the
caudate lobe of the unaffected twins liver. No such difference is found when
neither twin has hepachrinosis. From this information, it can be concluded
that hepachrinosis is caused by diminished liver capacity.
Which of the following is an assumption required by the argument?

A. Many diseases are caused by


diminished organ capacity.
B. The caudate lobe of a healthy identical
twin is the same size as the caudate
lobe of a healthy person who is not
an identical twin.
C. When two identical twins both suffer
from hepachrinosis, their caudate
lobes are the same size.
D. The relative smallness of the caudate lobe
of an affected person is not the result of
medications used in the treatment of
hepachrinosis.
E. People who have an identical twin are no
more likely to contract hepachrinosis than
people who do not have an identical twin.

500+ videos

800+ practice questions

step-by-step learning guide

All you need. All for free

Practice Question

(watch the entire video here)

Identical twins have livers that are genetically identical. Scientists recently
discovered that when one twin has hepachrinosis, a debilitating liver disease,
the caudate lobe of the affected twins liver is significantly smaller than the
caudate lobe of the unaffected twins liver. No such difference is found when
neither twin has hepachrinosis. From this information, it can be concluded
that hepachrinosis is caused by diminished liver capacity.
Which of the following is an assumption required by the argument?

A. Many diseases are caused by

P: Twins have = livers

diminished organ capacity.

B. The caudate lobe of a healthy identical

twin is the same size as the caudate


lobe of a healthy person who is not
an identical twin.

C. When two identical twins both suffer

P: Lobe of affected twin is smaller

P: Lobes of unaffected twins are =


C: Small lobe causes H
A: nothing else causes H

from hepachrinosis, their caudate


lobes are the same size.

D. The relative smallness of the caudate lobe


of an affected person is not the result of
medications used in the treatment of
hepachrinosis.
E. People who have an identical twin are no

more likely to contract hepachrinosis than


people who do not have an identical twin.

500+ videos

800+ practice questions

step-by-step learning guide

All you need. All for free

Practice Question
On average, when 100 non-pregnant women take the Pregunda home
pregnancy test, 10 of them will test positive. That is, the test will indicate
that the women are pregnant. Conversely, when 100 pregnant women
take the Pregunda home pregnancy test, 99 of them will test positive.
Therefore, if a group of women take the Pregunda home pregnancy test,
the vast majority of those who test positive will be pregnant.

The above argument is flawed primarily because it


A. does not take into consideration
the tests margin of error
B. suggests that non-pregnant
women are more likely to test
positive than pregnant women are
C. presupposes the proportion of
pregnant women in the group
D. assumes that the tests accuracy
is not affected by other factors
E. ignores the fact that women
who do not test positive may
be pregnant

500+ videos

800+ practice questions

step-by-step learning guide

All you need. All for free

Practice Question

(watch the entire video here)

On average, when 100 non-pregnant women take the Pregunda home


pregnancy test, 10 of them will test positive. That is, the test will indicate
that the women are pregnant. Conversely, when 100 pregnant women
take the Pregunda home pregnancy test, 99 of them will test positive.
Therefore, if a group of women take the Pregunda home pregnancy test,
the vast majority of those who test positive will be pregnant.

The above argument is flawed primarily because it


A. does not take into consideration

P: 10% of NP test positive


P: 99% of P test positive

B. suggests that non-pregnant

C: Group majority of
positive are pregnant

the tests margin of error

women are more likely to test


positive than pregnant women are

A: Test accuracy is constant

C. presupposes the proportion of


pregnant women in the group
D. assumes that the tests accuracy

is not affected by other factors

E. ignores the fact that women

who do not test positive may


be pregnant

500+ videos

800+ practice questions

step-by-step learning guide

All you need. All for free

Practice Question
Jesse: The H1 vaccine prevents people from contracting thramboxia, a
disease that can cause short-term paralysis. While thramboxia is an
unpleasant disease with paralysis lasting up to 3 months, the H1
vaccine is not worth getting since hundreds of people die each year
from the H1 vaccination.
Harjit: But thousands of people die each year as a direct result of
contracting thramboxia. So, in addition to preventing thramboxia, the
H1 vaccine prevents deaths.
Harjit responds to Jesse by
A. questioning the truth of a statement on
which Jesses conclusion is based
B. suggesting that Jesses argument
overlooks an important consequence
C. noting that Jesses argument contradicts
that which he is trying to prove
D. reinforcing Jesses conclusion before
suggesting a new conclusion

E. citing evidence that disproves the


evidence cited by Jesse in drawing
his conclusion

500+ videos

800+ practice questions

step-by-step learning guide

All you need. All for free

Practice Question

(watch the entire video here)

Jesse: The H1 vaccine prevents people from contracting thramboxia, a


disease that can cause short-term paralysis. While thramboxia is an
unpleasant disease with paralysis lasting up to 3 months, the H1
vaccine is not worth getting since hundreds of people die each year
from the H1 vaccination.
Harjit: But thousands of people die each year as a direct result of
contracting thramboxia. So, in addition to preventing thramboxia, the
H1 vaccine prevents deaths.
Harjit responds to Jesse by
A. questioning the truth of a statement on

P: H1 prevents T
P: T can cause paralysis
P: 100s die from H1

which Jesses conclusion is based

C: Dont get H1 vaccine

B. suggesting that Jesses argument


overlooks an important consequence

P: 1000s die from T


P: H1 prevents T & deaths

C. noting that Jesses argument contradicts

C: Get H1 vaccine

that which he is trying to prove

D. reinforcing Jesses conclusion before

suggesting a new conclusion

Theres also Y. My option


accomplishes 2 things

E. citing evidence that disproves the

evidence cited by Jesse in drawing


his conclusion

500+ videos

800+ practice questions

step-by-step learning guide

All you need. All for free

Practice Question
For most products there exists an inverse relationship between price
and consumption; if the price of a product increases, then consumption
of that product decreases, and if the price decreases, consumption
increases. However, in some poorer communities, when the price of
bread increases, the consumption of bread increases.
Which of the following, if true, does the most to explain the paradoxical
scenario?
A. Very few people living in poorer
communities are employed in the
bakeries that make the bread.
B. In poorer communities, there are very
few food alternatives to bread.
C. In poorer communities, bread is often
the least expensive staple food, so
when its price increases, people have
less money to spend on other, more
expensive foods.
D. Most people in poorer communities are
unlikely to change their eating habits.
E. During some holidays, the consumption
of food increases substantially.

500+ videos

800+ practice questions

step-by-step learning guide

All you need. All for free

Practice Question

(watch the entire video here)

For most products there exists an inverse relationship between price


and consumption; if the price of a product increases, then consumption
of that product decreases, and if the price decreases, consumption
increases. However, in some poorer communities, when the price of
bread increases, the consumption of bread increases.
Which of the following, if true, does the most to explain the paradoxical
scenario?
P: $ then buying
P: Bread $ then buying

A. Very few people living in poorer

communities are employed in the


bakeries that make the bread.

B. In poorer communities, there are very

few food alternatives to bread.

C. In poorer communities, bread is often


the least expensive staple food, so
when its price increases, people have
less money to spend on other, more
expensive foods.

Paradox: People buy


more bread when they
should buy less
Giffen good: A consumer
good for which
consumption increases
when price increases

D. Most people in poorer communities are

unlikely to change their eating habits.

E. During some holidays, the consumption

of food increases substantially.

500+ videos

800+ practice questions

step-by-step learning guide

All you need. All for free

Practice Question
It is very difficult for piano retailers to grow in the piano market. Despite
population increases, the demand for pianos has remained constant for
decades. Furthermore, when people decide to buy a piano, they typically go
to the nearest piano retailer to make their purchase. Given this environment,
a piano retailer can gain market share in the piano market only through
purchasing its competitors, and not through aggressive marketing strategies.
Which of the following, if true, contributes most strongly to a refutation of
the conclusion above?
A. Of the four mergers of piano retailers over the
past six years, three led to a decline in profits
for the newly-merged companies.
B. Through staff layoffs and warehouse closures,
some of the larger piano retailers have achieved
price reductions that are causing the smaller
piano retailers to go out of business.
C. Most piano retailers sell a variety of products
other than pianos.

D. Many public schools, traditionally responsible for


almost half of all piano purchases, are eliminating
their music programs in the face of severe budget cuts.
E. Several piano retailers have used aggressive marketing
strategies such as publicity events, direct mail and instore financing to increase customer loyalty.

500+ videos

800+ practice questions

step-by-step learning guide

All you need. All for free

Practice Question

(watch the entire video here)

It is very difficult for piano retailers to grow in the piano market. Despite
population increases, the demand for pianos has remained constant for
decades. Furthermore, when people decide to buy a piano, they typically go
to the nearest piano retailer to make their purchase. Given this environment,
a piano retailer can gain market share in the piano market only through
purchasing its competitors, and not through aggressive marketing strategies.
Which of the following, if true, contributes most strongly to a refutation of
the conclusion above?
A. Of the four mergers of piano retailers over the

past six years, three led to a decline in profits


for the newly-merged companies.

P: Hard for retailer to


grow
P: Demand constant
P: People buy from closest

B. Through staff layoffs and warehouse closures,


C: share only through
some of the larger piano retailers have achieved
buying competitors
price reductions that are causing the smaller
piano retailers to go out of business.
A: No other way to share
C. Most piano retailers sell a variety of products

other than pianos.

D. Many public schools, traditionally responsible for

almost half of all piano purchases, are eliminating


their music programs in the face of severe budget cuts.

E. Several piano retailers have used aggressive marketing

strategies such as publicity events, direct mail and instore financing to increase customer loyalty.

500+ videos

800+ practice questions

step-by-step learning guide

All you need. All for free

Practice Question
Maltania ranks far behind Finland and Sweden when it comes to employee job
satisfaction. In all three countries, revenue-sharing programs and onsite
daycare successfully increase job satisfaction. In Finland and Sweden, labor laws
require companies with more than 10 employees to have revenue-sharing
programs and onsite daycare. In Maltania, however, revenue-sharing programs
and onsite daycare are found in only a few companies that have voluntarily
established them.

Which of the following can be inferred from the statements above?


A. In Finland and Sweden, prior to the introduction
of labor laws, the level of employee job
satisfaction was similar to that in Maltania.
B. Revenue-sharing programs and onsite daycare
are no more effective in increasing employee job
satisfaction when they are established voluntarily
than when they are established to comply with
labor laws.
C. Establishing revenue-sharing programs and onsite daycare
at all Maltanian companies with over 10 employees would
increase employee job satisfaction in Maltania.
D. Companies with high job satisfaction typically have
revenue-sharing programs.
E. Prior to the introduction of labor laws in Finland and
Sweden, most companies with more than 10 employees did
not have revenue-sharing programs and onsite daycare.

500+ videos

800+ practice questions

step-by-step learning guide

All you need. All for free

Practice Question

(watch the entire video here)

Maltania ranks far behind Finland and Sweden when it comes to employee job
satisfaction. In all three countries, revenue-sharing programs and onsite
daycare successfully increase job satisfaction. In Finland and Sweden, labor laws
require companies with more than 10 employees to have revenue-sharing
programs and onsite daycare. In Maltania, however, revenue-sharing programs
and onsite daycare are found in only a few companies that have voluntarily
established them.

Which of the following can be inferred from the statements above?


A. In Finland and Sweden, prior to the introduction

of labor laws, the level of employee job


satisfaction was similar to that in Maltania.

B. Revenue-sharing programs and onsite daycare

are no more effective in increasing employee job


satisfaction when they are established voluntarily
than when they are established to comply with
labor laws.

P: M behind F&S in JS
P: RS & DC JS
P: F&S must have RS & DC
P: M has voluntary RS & DC

C. Establishing revenue-sharing programs and onsite daycare


at all Maltanian companies with over 10 employees would
increase employee job satisfaction in Maltania.
D. Companies with high job satisfaction typically have

revenue-sharing programs.

E. Prior to the introduction of labor laws in Finland and

Sweden, most companies with more than 10 employees did


not have revenue-sharing programs and onsite daycare.

500+ videos

800+ practice questions

step-by-step learning guide

All you need. All for free

Practice Question
Representative for the Maltanian Association of Cyclists: Over 1000
cyclists died in accidents last year. Accident reports indicate that 80%
of the victims were not wearing a helmet at the time of their accident.
This information indicates that, by wearing a helmet, cyclists can
reduce their risk of dying if they are involved in an accident.
Which of the following, if true, best supports the representatives
argument?

A) The Maltanian Association of Cyclists is


not funded by any helmet manufacturers.
B) Maltania has laws requiring all cyclists
to wear a helmet.
C) Last year, less than 20% of the cyclists
involved in accidents were not wearing a
helmet at the time of the accident.
D) Most of the accidents in Maltania involve a
cyclist and a motor vehicle.
E) Most of the cyclists who survived an
accident last year indicate that their injuries
would have been much worse had they not
been wearing a helmet at the time.
500+ videos

800+ practice questions

step-by-step learning guide

All you need. All for free

Practice Question

(watch the entire video here)

Representative for the Maltanian Association of Cyclists: Over 1000


cyclists died in accidents last year. Accident reports indicate that 80%
of the victims were not wearing a helmet at the time of their accident.
This information indicates that, by wearing a helmet, cyclists can
reduce their risk of dying if they are involved in an accident.
Which of the following, if true, best supports the representatives
argument?
+

A) The Maltanian Association of Cyclists is

P: 1000 cyclist deaths


P: 80% of victims no helmet

not funded by any helmet manufacturers. C: Helmets save lives


B) Maltania has laws requiring all cyclists

A: Less than 80% of cyclists


to wear a helmet.
in accidents were not
C) Last year, less than 20% of the cyclists
involved in accidents were not wearing a
helmet at the time of the accident.

wearing a helmet

D) Most of the accidents in Maltania involve a

cyclist and a motor vehicle.

E) Most of the cyclists who survived an

accident last year indicate that their injuries


would have been much worse had they not
been wearing a helmet at the time.

500+ videos

800+ practice questions

step-by-step learning guide

All you need. All for free

GMAT Critical Reasoning - Everything you need to know

For additional practice questions, see the


bottom of our Critical Reasoning module

www.GMATPrepNow.com

500+ videos

800+ practice questions

step-by-step learning guide

All you need. All for free

GMAT Critical Reasoning - Everything you need to know

If you enjoyed this learning format, let us


know, and well add similar resources to our
SlideShare page.

500+ videos

800+ practice questions

step-by-step learning guide

All you need. All for free

También podría gustarte